La gravité est-elle instantanée ou le café seulement ?


 

(Le présent article a maintenant une suite : kn0l.wordpress.com/la-gravite-est-elle-instantanee-ou-le-cafe-seulement-2/)

Celui qui s’adonne à la lecture d’ouvrages de vulgarisation sur la théorie de la relativité ou à celle des nombreux articles qui abordent également le sujet sera retombé…. comme une pomme ? mais non attendez un peu !… sera retombé des dizaines de fois sur les deux idées suivantes :

1 d’après la Théorie de la Relativité rien ne peut se déplacer, ou agir, plus vite que la vitesse de la lumière dans le vide…

Cette limite de vitesse est même souvent présentée comme un point central, ou le point de départ de la théorie de la relativité. En réalité, une recherche approfondie montre que son vrai axiome n’est pas que la vitesse limite EST partout la même, mais plutôt que la MESURE de la vitesse de la lumière donnera le même résultat partout dans l’univers, même si la source ou le récepteur se déplacent très vite par exemple. C’est la vitesse MESURÉE qui est la même seulement ! (ce qui est complètement différent)

2 DANS LE MÊME TEMPS et malgré la reconnaissance générale de la Relativité… eh bien au niveau de la course des astres, lunes et planètes notamment… « business as usual » (comme d’habitude) tous les manuels reconnaissent que les calculs de Newton RESTENT utilisables tant que les masses et les vitesses se maintiennent dans la limite du « raisonnable » (c’est à dire ne deviennent pas « relativistes »).

Dans le système solaire par exemple, seule la planète Mercure, qui passe assez vite et assez près du Soleil est affectée par des effets « relativistes » (extrêmement faibles d’ailleurs). C’est pour cette raison que le « décalage » de l’orbite de Mercure (précession du périhélie) non dû à la perturbation des autres planètes fut considéré comme une des premières preuves de la validité de la Relativité. (voir Remarque)

Ces deux idées sont évoquées sans gène dans les mêmes ouvrages, quoique toujours dans des chapitres différents, donc jamais réellement  confrontées.

Quand il s’agit d’un chapitre sur la lumière par exemple, la première règle sera souvent rappelée et ses effets seront détaillés par des exemples bien connus : comme la seconde qui sépare la voix de l’astronaute qui parle sur la lune et le moment où le technicien sur terre commence à l’entendre ( les ondes radios sont en fait une sorte de « lumière »)  ou encore le temps que met la lumière du soleil (8 mn et des brouettes) à nous parvenir.

Mais quand il s’agit de la gravitation, soit du contexte de la deuxième constatation, l’hypothèse centrale de la vitesse indépassable n’est plus évoquée frontalement, en général. On ne trouve pas de livre de cosmologie avec un chapitre commençant par les mots : « La gravité se propage à la vitesse de la lumière » (ou du même tonneau).

Galaxies lointaines, galaxies d’antan

Parlant d’astres VRAIMENT lointains, on a souvent droit à un rappel que ce que nous voyons n’est pas l’astre actuel mais cet astre vu ‘dans le passé ».

Par exemple pour une galaxie qui est située, si l’on en croit les calculs basés sur le décalage vers le rouge et la constante de Hubble, à 5 milliards d’années-lumière, ce que nous voyons, c’est cette galaxie telle qu’elle était il y a 5 milliards d’années bien sûr.

Ici on remarque un début de flou car les articles ne précisent généralement pas que la galaxie est vue non seulement « comme elle était » il y a 5 milliards d’années, OK, mais aussi LÀ OÙ ELLE ÉTAIT il y 5 milliards d’années, en éloignement notamment.

Donc, le décalage vers le rouge de la lumière provenant de cette galaxie est utilisé pour calculer sa vitesse apparente de « fuite » (par rapport à nous) et, voyez-vous, c’est cette vitesse qui sert dans un deuxième temps à évaluer sa distance.

Beaucoup d’articles et vidéos sont incroyablement légers car ils ne précisent pas bien qu’en conséquence la vitesse calculée sans précaution pourrait donc être la vitesse de cette galaxie IL Y A 5 milliards d’années et que la position reconstituée – d’après cette vitesse – ne pourrait aussi être que celle qui fut la sienne dans ce passé lointain.

Ensuite on nous dit invariablement et assez prestement que plus on observe des objets lointains plus leur vitesse de fuite est grande et que pour résumer, la vitesse est proportionnelle à la distance (comme ici http://www.podcastscience.fm/tag/relativite/). Si je reviens à ce que je viens d’écrire plus haut, il faudrait se rappeler que plus les objets sont lointains, plus ils sont anciens, et plus leur vitesse estimée est périmée !

Ainsi par exemple, sur la page : http://www.perseus.gr/Astro-DSO-QSO-3C273.htm, on lit : »Its redshift (z=0.158339) suggests that it is receeding away from us at 14.7% the speed of light (ie. 44,000 km/sec)!« . L’auteur oublie (?) qu’il vient juste d’écrire que « it is one of the most distant objects visible with a backyard telescope at a distance of 2.18 billion light-years away » donc le quasar en question n’est pas EN TRAIN (receeding est un progressif) de s’éloigner à 14% etc., mais plutôt : Il ÉTAIT en train de le faire IL Y A 12 milliards d’années ( au moins?).

Dans un univers en expansion ‘élastique’ où toute distance serait multipliée par le même coefficient par unité de temps et où la lumière serait instantanée, il me semble que la vitesse observée serait bien proportionnelle à la distance mais si on remplace maintenant, dans le même univers, la lumière instantanée par une lumière qui voyage à la vitesse c, on ne devrait plus trouver cette proportionnalité vous ne croyez pas ?

Les astres lointains devraient avoir en toute logique un décalage moins important que leur distance réelle car ils sont vus dans le passé, quand ils avaient une vitesse moindre. Il est vrai que l’espace, sur le trajet que la lumière a parcouru pendant ces milliards d’années, a continuer à se dilater. Mais si le décalage vers le rouge est réellement lié à cette dilatation, il n’est plus le signe d’une vitesse (par effet Doppler) mais le résultat de l’intégration de cette vitesse pendant une durée.

En écrivant ces lignes je n’ai fait que m’apercevoir que la plupart des exposés pédagogiques sur le Red Shift et l’expansion de l’univers sont prévus pour des gens qui n’ont aucune base mathématiques ou physiques. Autrement dit, dans le but de rester compréhensibles, ils escamotent la réalité mathématique du sujet pour la remplacer par une réalité plus ‘pédagogique’. Une petite recherche permet de fait de rétablir la vérité assez aisément : https://fr.wikipedia.org/wiki/Loi_de_Hubble#Modifications_.C3.A0_la_loi_de_Hubble

La lumière aberrante

Il faut fouiller déjà pas mal sur ces sujets pour commencer à entendre parler d’un phénomène plus confidentiel évoqué sous le nom curieux « d’aberration de la lumière », quelquefois « abberation stellaire ». Dans la présentation de Wikipedia, le texte a dû être rédigé à l’époque pré-relativiste car il précise que c’est un phénomène dû à la vitesse de l’observateur…. https://fr.wikipedia.org/wiki/Aberration_de_la_lumi%C3%A8re

Or bien sûr dans l’univers, toute vitesse est relative. La vitesse de la terre par exemple est et ne peut être définie que par rapport au Soleil (en général). Par rapport à notre galaxie, la vitesse de la terre devient négligeable et elle devient égale en moyenne à celle du soleil.

Bien entendu, l’aberration stellaire n’est pas due « à » une vitesse mais bien au fait que la terre « tourne autour du soleil »… c’est à dire plus exactement que cette vitesse « change de direction ».

Mais rappelez-vous? la terre, elle « tourne » à cause de la gravité, pas à cause de la lumière..

Or la gravité n’est pas citée dans l’article, comme je le constatais plus haut (on parle soit de l’un soit de l’autre) ce qui pourrait dans le cas contraire provoquer des questions gênantes peut-être…comme : « il y a-t-il une aberration de la gravité ? »

(Pour mettre un peu d’eau dans mon vin, lors de l’écriture de cet essai, il m’a semblé remarquer que la relativité générale était de plus en plus souvent présentée dans les écrits modernes comme une « théorie de la gravitation », alors que j’étais resté sur l’impression (vraie ou fausse) que dans le passé, elle était présentée davantage comme une théorie cosmologique, sans référence spéciale à la gravitation (disons un théorie qui intégrait la gravitation, mais pas comme une théorie DE la gravitation ?)).

LA FAILLE LOGIQUE

Comme on vient de le voir, les deux sujets, celui de la lumière, enfin … de sa vitesse limite … et celui de la gravité sont toujours traités séparément. Un autre artifice qui est utilisé pour garder chacun chez soi est que la vitesse limite de TOUT dans l’univers reste quand même présentée sempiternellement comme la vitesse « DE LA LUMIÈRE » (au pire, des ondes radios, et dans quelques articles, de « l’information ». On ne lit pas de : « dans l’univers la vitesse limite est celle de la gravité » mais on verra que le vulgarisateur moyen s’empresse de le conclure).

Bien sûr, cette vitesse limite s’applique censément aussi aux ondes électromagnétiques en général et même à la vitesse des objets mais elle reste en général exprimée sempiternellement comme une vitesse DE LA LUMIÈRE !

Revenons sur la théorie de la gravitation de Newton. L’histoire de la pomme est souvent très mal racontée, malgré les nombreuses tentatives pédagogiques de Gotlib en particulier. (Il vient de nous quitter, Gotlib, pas Newton)

newton.aspx_

Notez que Newton décolle un peu du sol, attiré par la pomme ! J’ai une question : pourquoi la coccinelle est-elle fâchée ?

C’est pourtant simple (mais rarement bien expliqué): Newton est le premier homme qui a compris que la chute banale d’un objet comme une pomme et la révolution de la lune autour de la terre n’étaient qu’un seul et même phénomène. De plus il a correctement théorisé la symétrie de l’attraction, encore mal visualisée par beaucoup d’auteurs non-scientifiques (qui écrivent que la Terre exerce une force sur la pomme et que la pomme accélère vers la terre, mais négligent que la pomme exerce la même force sur la Terre).

On présente souvent sa formule qui donne la force d’attraction entre deux masses (donc grosso modo entre deux corps) même à des étudiants peu avancés en physique tellement elle est simple. La « force » d’attraction newtonienne (en réalité une double force) est juste proportionnelle aux masses en jeu (donc au produit des deux masses dans le modèle le plus simple : masses ponctuelles) et inversement proportionnelle au carré de la distance (de ces masses ponctuelles). (calcul avec la somme des masses ici lol)

On peut difficilement trouver une formule plus simple en effet ! La proportionnalité inverse au carré peut effrayer un peu les allergiques aux maths mais cela revient juste à dire que la gravité se « répand » autour de sa source avec un potentiel d’action qui diminue dans la même proportion que la surface touchée augmente (la surface de la sphère est proportionnelle au carré du rayon).

Pour prendre une comparaison dans le bricolage, science des hum, moins intellectuels… si avec le même pot de peinture on voulait peindre des ballons de plus en plus gros, l’épaisseur de la couche de peinture diminuerait comme la surface des ballons, soit comme leur rayon AU CARRE, exactement la même progression mathématique que la gravité !

C’est aussi le même calcul ‘inversement proportionnel à la distance’ qui donne l’énergie lumineuse qui parvient à une surface donnée qui s’éloigne d’une source et qui fait que si vous allez deux fois plus loin de la lampe, vous serez quatre fois moins éclairé, et trois fois plus loin, neuf fois moins éclairé :

Si, faisant passer la lumière d’une bougie par un petit trou, vous placez à un pied de distance la surface A d’un pouce carré, cette surface jettera sur B qui est à deux pieds, une ombre de quatre pouces carrés ; sur E qui est à trois pieds, une ombre de neuf pouces ; sur D qui est à quatre pieds, une ombre de seize pouces ; sur cinq, une ombre de 25 ; sur six, une ombre de 36 ; en un mot l’ombre augmentera comme le carré des distances. CONDILLAC

1-Kel cannaweed

image trouvée par une recherche ouverte et qui s’avère venir du site Cannaweed ! Que font-ils avec leur bougie, ils allument leur joint ou ils éclairent leurs cultures dans leur placard ?

À force de se concentrer sur l’intensité de la gravitation et son calcul, notamment par la phrase beaucoup répétée « Deux corps quelconques s’attirent selon une force proportionnelle au produit de leur masse et inversement proportionnelle au carré de la distance qui les sépare » on parle moins de sa représentation dans la théorie de Newton : les deux masses sont immobiles !

CAPISCE ? Newton a établi sa loi avec une gravité immédiate mais cela ne se voit pas dans la formule ! . Qu’en est-il si ces masses sont en mouvement ?

Ce modèle est bel et bon quand les deux masses sont proches mais « à la lumière » de la Théorie de la Relativité, une fois de plus, rien n’est censé se déplacer, ou se propager, à une vitesse supérieure à c donc dès que la distance augmente suffisamment, on devrait observer un retard de la gravité dans son orientation dès que les masses se déplacent l’une par rapport à l’autre, et aussi peut-être une aberration, qu’en pensez-vous ?

Arrivés là, soit vous attachez la plus grande importance au point 1 et vous vous mettez à affirmer à qui veut l’entendre qu’étant donné que dans la Relativité Générale, TOUT, je dis bien TOUT messieurs dames, ne peut pas dépasser

‘la vitesse que la lumière elle dépasse pas’

et que donc la gravité ne peut échapper à ce principe de base… parole d’évangile.

Soit vous avez des notions d’astronomie et vous savez qu’avant Newton, après Newton et après Einstein, les astres ont continué à tourner de la même façon bête et régulière. Autrement dit vous savez que le point 1 est une théorie, mais que le point 2 (du début de cet article) représente un fait : les planètes tournent autour du Soleil en suivant les lois de NEWTON !

Donc si du temps de Newton sa loi sur la gravitation universelle ET INSTANTANÉE permettait déjà de calculer correctement la trajectoire des planètes, il est clair que si on introduit dans les calculs, un freinage ou une accélération tangentielles dus à la propagation non-instantanée de la gravitation quelque chose d’embêtant va se passer.

On reviendra plus loin sur cette idée.

ETAT DES LIEUX D’UN DÉBAT PEU DÉBATTU

Maintenant que j’ai posé le cadre du problème, je vais étudier ce qu’il en ressort à tous les niveaux de la vulgarisation disponible.

Mon étude se fera sur les documents/sites disponibles sur le Web. Je comprends que ce n’est pas entièrement de la littérature officielle, mais tant mieux. Mon raisonnement est le suivant :

1 étant donné que les chercheurs qui travaillent dans les différentes institutions et universités etc. font des publications

2 que ces publications sont vulgarisées par des journalistes scientifiques

3 que toute cette info fini par descendre vers le public et ce, jusque dans les forum. (fora ?)

4 le WWW est donc un bon moyen, surtout en anglais on le verra, de connaître l’état de l’art sur la question d’autant qu’il peut souvent permettre de remonter aux publications initiales en cas de doute.

Au pire, si on détecte une question litigieuse dans un document de « basse vulgarisation », on pourra donc s’élever vers des sources plus « sérieuses » pour voir comment elle y est traitée…

En français

Commençons par cette page de Wikipedia :

https://fr.wikipedia.org/wiki/Gravitation

Très rapidement, on peut y localiser une confusion étonnante :

« La théorie ainsi construite, qui porte le nom de relativité générale, incorpore le principe de relativité, et la théorie newtonienne en est une approximation dans la limite des champs gravitationnels faibles et des vitesses petites devant celle de la lumière. En effet, les déformations de l’espace-temps prévues sous l’effet des corps massifs, quand ceux-ci ont une forte accélération, ne se propagent pas plus vite que la vitesse de la lumière, ce qui résout le paradoxe de l’instantanéité apparente de l’interaction newtonienne. Il en résulterait des ondes gravitationnelles, qui restent encore à observer »

En l’espace de quelque mots, on observe comment le rédacteur saute de la notion de champ gravitationnel (qui est le sujet de la page) à celui d’ondes gravitationnelles !

J’ai coloré en vert les morceaux de la phrase qui parle de la Gravitation et en rouge ceux qui parlent (ou semblent dériver vers) les ondes gravitationnelles (la gravitation en effet, ne se soucie pas des accélérations).

Par curiosité, allons donc  à « ondes gravitationnelles » :

https://fr.wikipedia.org/wiki/Onde_gravitationnelle

« Dans la théorie de la relativité générale, la gravité provient de la courbure de l’espace-temps. Cette courbure est causée par la présence d’objets possédant une masse. Plus la masse de l’objet est grande, plus la courbure produite est grande et ainsi plus la gravité est intense. Lorsque des objets massifs se déplacent dans l’espace-temps, la courbure de l’espace-temps s’ajuste pour refléter le changement de la position de ces objets. Sous certaines circonstances, les objets accélérés peuvent produire une perturbation de l’espace-temps qui s’étend et se propage de manière analogue à « des vagues à la surface de l’eau ». On désigne par onde gravitationnelle ce type de perturbation. On prédit qu’elles se propagent à la vitesse de la lumière. Ainsi, l’existence des ondes gravitationnelles résulte en quelque sorte de l’application à la gravité du principe d’invariance de Lorentz qui introduit le concept de vitesse limite pour la propagation des interactions physiques (concept inexistant dans la vision newtonienne de la gravitation, cette interaction se propageant à une vitesse infinie dans cette théorie).

L’analogie entre des charges électriques en mouvement et des masses en mouvement permet de mieux appréhender le phénomène : de la même manière que l’accélération de particules chargées produit des ondes électromagnétiques, l’accélération de particules possédant une masse produit des ondes gravitationnelles. La plupart des théories de gravité quantique postulent l’existence d’un quantum correspondant appelé le graviton de façon analogue à l’électrodynamique quantique dans laquelle le vecteur de la force électromagnétique n’est autre que le photon. L‘onde gravitationnelle est associée au graviton et ses caractéristiques donnent alors de précieuses informations sur cette particule.« 

Remarquez cette phrase bizarre : «  Lorsque des objets massifs se déplacent dans l’espace-temps, la courbure de l’espace-temps s’ajuste pour refléter le changement de la position de ces objets ». La courbure ne semble pas se PROPAGER ! Elle s’AJUSTE, un verbe bien rare dans l’environnement physico-mathématique que cette prose est censée vulgariser.

La même personne, ou une autre, s’est mise légèrement en porte à faux avec cette affirmation peu orthodoxe. Heureusement la suite du texte, très caractéristique, rétabli l’équilibre : on commence par dire que « sous certaines conditions » des ondes gravitationnelles sont créées… elles se déplacent, à la vitesse c…. puis tout à coup on ne parle plus des ondes gravitationnelles, mais de la GRAVITE, et de la gravitation newtonienne….. ensuite la gravité (pas les ondes gravi, évidemment) seraient dues au graviton, et pof on reparle de la vitesse de la lumière.

À l’issue de ces deux paragraphes, si le lecteur n’a pas la tête qui tourne, c’est qu’il n’a pas essayé de comprendre. S’il a essayé de comprendre, il est comme celui qui regarde le bonneteau. Comme le sujet est difficile et qu’on ne lui a pas vraiment dit clairement que la gravité (courante, mesurée) et que les ondes gravitationnelles (rares et non encore mesurées) étaient deux choses différentes et qu’ensuite on a parlé tantôt de l’une tantôt de l’autre, c’est normal !

bosch-shell2

Bien sûr, vous me direz « ah Wikipedia, ha ha, Wikipedia, ha ha ha, mais Wikipedia n’a aucune valeur tout le monde sait ça« .

Oui, je sais, tout le monde consulte Wikipedia, mais personne ne l’admet. En plus il est connu que les pages mathématiques et physiques de WP sont celles qui sont rédigées par les charlatans les plus enragés (non, simple plaisanterie, ce sont les pages les plus sérieuses normalement, car personne ne s’y risque).

Sautons donc à un site sur l’astrophysique, la cosmologie, tout ça :  http://www.astrosurf.com/luxorion/index.htm

C’est le site Luxorion, il aurait 1800 pages (avouées, je n’ai pas vérifié)

La page de ce site concernant les ondes gravitationnelles est : http://www.astrosurf.com/luxorion/relativite-generale-ex5.htm

(désolé, je repars de ces ondes QUI NE SONT PAS LA GRAVITATION, mais c’est pour montrer que c’est de ces ondes hypothétiques que viennent les fausses idées sur la gravitation, pas forcément exprimées aussi explicitement sur une page titrée « gravitation », qui la plupart du temps, n’abordera pas de fait la question de la vitesse, comme on l’a posé plus haut)

Toute la page reste bien focalisées sur les Ondes Gravi, mais vers la fin, subitement on se met à lire que :

«  Kepler avait l’intuition que la force de la gravitation agissait comme l’intensité de la lumière, avec une force inversement proportionnelle au carré de la distance. Newton, comme Descartes, n’acceptaient pas son influence instantanée mais sans pouvoir l’expliquer, ils durent s’y plier et ce concept fut dogmatisé jusqu’aux travaux de Maxwell. Si l’onde gravitationnelle se propage à une vitesse finie, elle influencera la position instantanée des deux étoiles d’une quantité liée au temps qu’elle mettra pour se propager d’une étoile à l’autre, induisant une accélération ou une décélération du mouvement orbital. Dans l’exemple du pulsar binaire, l’accélération observée ne diffère que de 4% par rapport à la valeur théorique déterminée par la relativité générale. La théorie d’Einstein confirme son exactitude, y compris dans les conditions extrêmes de champs variables (de propagation). On détermina également que la vitesse de propagation de la gravité coïncidait avec la vitesse de la lumière avec une précision de 1%.« 

Donc à la fin de cette page consacrée aux OG l’auteur revient brusquement sur la gravitation elle-même, chez Newton, puis repart tout aussi brusquement sur les ondes G, mais voilà t-y pas qu’elles se mettent à « influencer la position instantanée des deux étoiles« , à « induire une accélération ».

Il est pourtant clair d’après  la théorie tout ce qu’il y a de plus orthodoxe, que les ondes G ne peuvent accélérer un corps ou « influencer » sa position instantanée, c’est la gravité qui fait ça pas les ondes gravitationnelles !

L’auteur s’est aussi mélangé dans l’ordre de son raisonnement logique :

1 on détecte un ralentissement de la révolution de ce pulsar,

2 SI ON L’ATTRIBUE à l’émission d’OG (non détectées à ce jour, c’est fait depuis, correction de déc.2016)

3 ALORS ce ralentissement colle avec les calculs relativistes.

Mais bien évidemment, ce ne sont pas les OG qui CAUSENT le ralentissement, pas plus que le bruit dans les arbres ne cause le vent. Embourbé dans cette erreur, l’auteur pourrait avoir l’idée d’en sortir. Au lieu de ça il entérine définitivement la confusion en concluant sa page censément sur les ondes gravitationnelles en parlant de la « propagation de la gravité ».

Assez perdu de temps, nous avons besoin besoin d’une référence plus sérieuse. La RECHERCHE, ça c’est du lourd !

http://www.larecherche.fr/idees/back-to-basic/gravitation-01-11-1997-88400

L’article n’est malheureusement pas signé, ça commence mal pour du sérieux ! (il semblerait d’après http://jac_leon.perso.neuf.fr/gravitation/article-francais/f-1.html que l’auteur  en soit Stéphanie RUPHY, journaliste à La Recherche, avec la collaboration de Jean-Marie Lévy-Leblond, professeur de physique à l’université de Nice.

On peut lire les deux premiers paragraphes sur l’attraction sans s’apercevoir que la Terre attire la lune, mais que la lune (ou n’importe quel corps) attire aussi la terre, donc que la lune « tombe » sur la terre, si on veut, mais que la Terre tombe aussi sur la lune en fait. Ce biais est très mauvais pédagogiquement, on le verra dès la suite de l’article.

Dans cette suite on apprend que l’on sent notre propre poids « en montant un escalier », je suis heureux de l’apprendre et je plains les gens qui n’ont pas d’escalier, et qui de ce fait ne peuvent sentir leur poids, merci LA RECHERCHE !

Mais l’article se lance dans l’explication des marées, c’est tellement fort que je ne peux que reproduire ce passage :

« L’attraction gravitationnelle mutuelle de la Lune et de la Terre tend à les faire se rapprocher l’une de l’autre. Mais cette attraction est compensée par la force centrifuge de rotation [c’est pas la révolution plutôt ?] de la Terre, comme de la Lune, autour de leur centre d’inertie point G sur le schéma ci-dessous [on n’a pas le schéma mais on essaie de suivre quand même, surtout qu’il s’agit du fameux point G ! 🙂 »]. Au centre de la Terre, la force centrifuge et la force d’attraction exercée par la Lune se compensent [ça alors, juste au centre ?]. Mais ce n’est pas le cas en un point quelconque de la surface terrestre car les deux forces varient en sens contraire : plus un point est éloigné du centre de gravité Terre-Lune, plus la force centrifuge qu’il subit sera grande, alors qu’au contraire l’attraction gravitationnelle exercée par la Lune décroît avec la distance. Les deux forces ne se compensent donc pas à la surface de la Terre et leur différence est à l’origine des marées : au point A, la force centrifuge est insuffisante pour contre-balancer l’attraction gravitationnelle, A va donc tendre à se déplacer vers la Lune. Inversement, au point B la force centrifuge est plus grande que la force exercée par la Lune et B va donc tendre à s’en éloigner. Voilà pourquoi il y a sur Terre une marée deux fois par jour [vous êtes sûr qu’il y a toujours DEUX marées partout « sur terre », c’est  pas plutôt sur mer ?]. Ce phénomène d’attraction différentielle affecte l’ensemble de la surface terrestre [pas l’intérieur ? ], mais seule la déformation des océans est facilement perceptible, la croûte terrestre étant trop rigide pour que sa forme soit significativement altérée. Cette déformation s’accentue lorsque le Soleil est aligné avec la Lune et la Terre et ajoute alors son effet de marée propre. C’est donc à la pleine Lune et à la nouvelle Lune que les marées sont les plus spectaculaires [et à l’équinoxe non ?]. »

Voilà ce qui arrive quand on veut expliquer les marées avec une vision statique, si l’auteur était parti de l’attraction de la lune sur la terre, qui engendre une accélération différentielle selon la distance, en vertu de la loi de Newton, il serait arrivé directement à l’idée d’une déformation !

En passant, il lui serait peut-être resté de la place pour souligner qu’à ce stade, c’est la rotation de la terre qui provoque le déplacement de cette contrainte (sans rotation pas de marées, s’il y avait des mers sur la lune, elles n’auraient pas de marée terrestre, seulement une très lente marée solaire), et que c’est à leur tour la forme des bassins qui détermine l’amplitude de la marée observée (qui n’est pas toujours bi-diurne monsieur-et-madame-qui-passez-vos-vacances-en-Bretagne-depuis-toujours).

Passons sur les 1057 atomes dans le soleil, il s’agit bien sûr d’une coquille pour 10 puissance 57 soit 1057, mais je rappelle que c’est un journal sérieux, que la page est publiée depuis novembre 1997, soit presque vingt ans. Certains lecteurs doivent toujours se demander comment les 1057 atomes du soleil sont rangés ?

Toutes les considérations qui viennent ensuite sur le rôle de la gravitation dans le destin et le fonctionnement d’une étoile me paraissent bonnes. On ne peut pas en dire autant sur la traduction de la phrase de Newton : « Hypotheses non fingo ». « Je ne feins », ça ne veut rien dire. Je n’émets pas d’hypothèse, ou je ne forge pas d’hypothèses, sont des traductions correctes.

Je ne suis pas historien des sciences, mais je ne crois pas que l’auteur a raison de dire qu’Einstein a voulu remplacer la théorie de Newton À CAUSE du problème de l’action instantanée, comme il l’affirme, cela pourrait être seulement à cause de l’action à distance, comme on le verra.

Q.Newton et Einstein sont-ils parfois d’accord ?

R.La relativité générale constitue une théorie de la gravitation qui englobe la théorie newtonienne : lorsqu’on résout les équations d’Einstein par approximations successives, on retrouve d’abord les résultats de Newton. [ah bon, donc l’instantanéité de l’attraction gravitationnelle ?]

Conscient ou pas de cette idée saugrenue qui pourrait surgir immédiatement, l’auteur se dépêche de sauter directement aux trous noirs et au périhélie de Mercure, qui est la balle de match, comme chacun sait, du tournoi Newton-Einstein (Einstein a renvoyé Mercure en lob juste au dessus du soleil et Newton a couru désespérément mais n’est pas arrivé à la rattraper !)

Quant à la théorie newtonienne, elle demeure le cadre d’étude de la plupart des phénomènes astronomiques. [comme le système solaire, ou les galaxies, avec leur attraction apparemment instantanée ?]

Le dernier paragraphe s’appelle « Comment se propagent les effets gravitationnels ? » Notons le le très ambigu « effets gravitationnels ».

On le voit, la lecture de La Recherche ne donne pas des informations beaucoup plus sérieuses que Wikipedia ou un site spécialisé.

Dans cette autre page de Wikipedia, on retrouve une rédaction curieusement semblable à ce que nous avons vu précédemment : https://fr.wikipedia.org/wiki/Critiques_de_la_th%C3%A9orie_de_la_relativit%C3%A9

« S’appuyant sur un argument de Pierre-Simon de Laplace (1749-1827), Poincaré (1904) fait allusion au fait que la loi de Newton sur la gravitation universelle est fondée sur une vitesse infinie de la gravité, de sorte que la synchronisation des horloges à l’aide de signaux lumineux pourrait être remplacée, en principe, par une synchronisation à l’aide de signaux gravitationnels instantanés. Mais en 1905, Poincaré résout ce problème en montrant que, dans une théorie relativiste de la gravitation, la vitesse des ondes gravitationnelles est égale à la vitesse de la lumière. Bien que sous une forme beaucoup plus compliquée, c’est aussi le cas dans la théorie de la relativité générale d’Einstein » [remarquer encore une fois le passage subliminal de la gravité elle-même aux ondes gravitationnelles]

[[dans la première rédaction de cet article, j’ai cru que dans ce passage, il y avait le classique passage de la gravité aux ondes gravitationnelles, comme vu plus haut. Mais depuis je me suis aperçu que c’est une autre confusion en jeu : Poincaré, lorsqu’il employait « ondes gravitationnelles » ( ou plutôt « ondes gravifiques » ?) parlait EFFECTIVEMENT de la gravité, et de sa propagation. C’est la transposition de ce qu’il a écrit avec un nouveau sens pour « ondes gravitationnelles » (au sens d’Einstein) qui provoque la confusion. Au passage, Poincaré n’a jamais pu démontrer sa vitesse c pour les ondes gravifiques à ma connaissance]]

CERTAINS SITES, si on a la chance de tomber sur eux, donnent une version plus précise, même si pas complètement honnête :

https://fr.wikipedia.org/wiki/Champ_gravitationnel#Aspects_historiques_:_loi_universelle_de_la_gravitation nous présente assez naïvement la notion de champ (dans le texte, pas dans les formules) et selon l’auteur, grâce au champ, on se libère du problème de l’instantanéité, car (d’après ce rédacteur) : « Le champ ou champ de force de la gravitation apparaît ainsi comme une propriété de l’espace due à la masse d’un corps. Une autre masse entrant en contact avec ce champ est soumise à une influence, une force, due au champ. Ainsi, l’influence gravitationnelle n’est pas, dans ce cadre, créée et transportée instantanément d’un corps à l’autre, mais est déjà présente dans tout l’espace sous la forme du champ et à son contact un corps voit sa dynamique modifiée.« 

L’auteur ne parait pas conscient que le corps B qui se balade et se retrouve tout à coup (« entre en contact ») dans un champ A qui était déjà là (« présent dans tout l’espace », et qui voit donc sa « dynamique modifiée » a lui aussi un champ B autour de lui ??? Comment se balade ce champ ?

Plus loin on lit que « À la limite des champs faibles, les équations du champ gravitationnel se ramènent à celles du champ classique newtonien. » Donc en clair, cela veut dire que la gravitation, champêtre ou pas, est bien calculée comme instantanée.

Un dernier site, pour la route ? http://astronomia.fr/6eme_partie/RelativiteRestreinte

C’est le cours d’astronomie générale de l’observatoire de Marseille, oui Marseilleu, et alore ? Qu’é-ceu queu vous lui reprochez à Marseilleu ?

Dans la page citée, on lit deux déclarations contradictoires :

« La Mécanique Classique admettait les interactions à distance instantanées (la gravitation en particulier est une force ne dépendant pas du temps pour sa transmission, mais attention, gravitation et lumière ne sont pas la même chose) »

Ici, claire distinction entre la lumière et la gravitation et emploi du présent pour l’instantanéité de la gravité qui contraste avec ce que la mécanique classique admettait, donc au passé. Mais un peu plus loin :

« Basée là-dessus, la Relativité Restreinte a abouti à une formalisation de l’espace-temps qui étend cette vitesse limite à toutes les interactions (gravitationnelle, faible, forte). Comment les propriétés de la lumière peuvent-elles s’étendre aux autres interactions ? En fait, la Relativité Restreinte décrit une propriété de l’espace-temps, non de la lumière. »

Là encore, un cas typique de schizophrénie : l’auteur sait que la gravitation est toujours instantanée, mais cela produit chez lui une dissonance cognitive. La gravité ne PEUT être instantanée puis que la relativité à posé que rien ne pouvait l’être. La seconde phrase vient recouvrir la première en la contredisant.

Maintenant que nous avons vu comment raisonnent les encyclopédies et autres haut-lieux de la vulgarisation, intéressons nous au degré zéro de la science, tout en bas de l’échelle pédagogique : les blogs et forums.

Sans surprise, on retrouve les mêmes approximations et raisonnements bancals. Sur les blogs, ils s’agit juste de reproduire en troisième main des affirmations qui viennent de plus haut, de la Recherche par exemple (l’article de la recherche étant lui-même un brouet issu d’une soupe plus concentrée concoctée dans tel ou tel laboratoire).

Exemples de blogs :

http://blog.slate.fr/globule-et-telescope/2012/12/27/la-gravite-se-deplacerait-bien-a-la-vitesse-de-la-lumiere/

http://www.gurumed.org/2012/12/29/einstein-avait-encore-raison-la-vitesse-de-la-gravit-atteindrait-bien-celle-de-la-lumire/

Sur les forums, schématiquement, cela se passe de la manière suivante :

Un pékin débarque naïvement sur un forum dédié à la physique ou à l’astronomie etc. et pose ingénument la  question de l’instantanéité de la gravitation, sous une forme plus ou moins imagée (en général : si le soleil disparaissait, ou bien « si le soleil était changé en trou noir » etc.). Aussitôt un surveillant de forum, aussi appelé « petit chef wikipédien » arrive avant que des débats ou des démangeaisons grattogènes puissent contaminer d’autres membres de la communauté, et assure sur un ton définitif au malheureux égaré que la gravitation se propage, bien sûr, à la vitesse de la lumière, comme chacun sait. Baisser de rideau.

http://forum.scienceamusante.net/viewtopic.php?t=871 (ici, l’intervenant qui explique que l’indépassabilité de la vitesse de la lumière est un axiome finit en concluant que la gravité DOIT se propager comme la lumière)

http://forums.futura-sciences.com/physique/629785-vitesse-de-gravite.html (ici après une réponse cinglante par Maxwellien, le forum finit par poser des vraies questions avec ansset, si un lecteur a le courage d’aller jusque là)

http://www.webastro.net/forum/showthread.php?t=14143

http://forum.hardware.fr/hfr/Discussions/Sciences/gravitation-discrete-continue-sujet_58555_1.htm

autres approches :

http://www.cerimes.education.fr/e_doc/forces/gravitation.htm Ici le problème est exposé de façon historique, Einstein a voulu remédier aux défauts de la théorie de Newton, ce qui n’empêche pas l’auteur de vivre sa schizose en affirmant à un moment que « La loi de la gravitation universelle de Newton est toujours utilisée pour calculer les effets de la gravitation dans des situations normales » bien que plus loin : « A l’heure actuelle, la relativité générale est la seule théorie capable de décrire tous les effets gravitationnels observés. » Le seul problème qui semble rester à la Relativité générale d’après cette page est son caractère non-quantique, tout le reste étant résolu…

https://fr.wikipedia.org/wiki/Histoire_de_la_gravitation#Entre_Newton_et_Einstein Ici la vision est devenue historique, cela a des inconvénients, on ne comprend plus toutes les péripéties si on est plutôt scientifique ! Cette page est typique de celles qui propagent la fable que l’action à distance posait d’autant plus un problème qu’elle était instantanée (« Une hypothèse époustouflante est que l’action à distance est instantanée. Newton lui-même se refuse à commenter ce fait …. »)

https://fr.wikipedia.org/wiki/Champ_gravitationnel, confirme la même fable, en citant pourtant la phrase de Newton qui l’infirme (voir plus bas)

Remontons vite à un plus haut niveau :

http://www.futura-sciences.com/magazines/matiere/infos/dossiers/d/physique-relativite-generale-espace-temps-devint-dynamique-510/page/2/ ce blog est écrit par un « docteur en physique théorique » et que lisons nous ? « Cependant, même s’il eut l’intuition que la vitesse de propagation de la gravitation était probablement égale à « c » et qu’il existait peut-être des « ondes gravifiques »2, Poincaré…

2 on parle aujourd’hui plutôt « d’ondes gravitationnelles« 

donc visiblement, l’auteur confond lui-aussi gravitation et ondes gravitationnelles, ou du moins les rapproche dangereusement dans une même phrase ce qui pourrait induire chez certains critiques malintentionnés l’idée qu’il passe de l’un à l’autre sans précaution…

Voilà une page avec une adresse qui « fait sérieux » :

http://www.aim.ufr-physique.univ-paris7.fr/CHARNOZ/homepage/GRAVITATION/grav7.html

On y lit :

« Une des conséquences les plus étonnantes de la relativité générale est la présence dans l’univers d’ondes gravitationnelles. En effet, une déformation de l’espace temps, induite par la présence d’un corps, ne disparaît pas comme ca. Elle se propage, à la vitesse de la lumière, comme une onde sonore ou lumineuse. » [sonore, really ?]

mais plus loin : « C’est une conséquence de la déformation de l’espace-temps induite par la formidable densité du trou noir. Toute l’information est perdue, sauf , la masse, le moment angulaire, et la charge électrique. »

Donc les ondes gravitationnelles « déforment » l’espace temps, et la gravité « déforme » l’espace-temps ? Curieuse ressemblance ??? Par ailleurs l’auteur (et il n’est pas isolé) oublie que la gravité du trou noir n’est pas perdue, disons que c’est ce qu’il sous-entend, on est d’accord, par sa « masse », mais il conviendrait de le préciser, puisque la masse « reste » dans le trou, mais que la gravité en sort, elle,  du trou…

Finalement, c’est tout à la fin de la page dans les exercices, au paragraphe intitulé « le retour de la comète de Halley », qu’on lira cette phrase très caractéristique : « Newton lui-même n’apportait pas d’explication à la nature de cette force, qui agit de manière instantanée entre tous les corps (comme on le croyait à l’époque, mais on sait maintenant qu’elle se transmet à la vitesse de la lumière).« . Si elle n’agit plus instantanément, monsieur le rédacteur, ou ne l’a jamais fait, il faudrait alors employer un autre temps que le présent de l’indicatif, comme l’imparfait « agissait » ou d’autres formes verbales exprimant un doute « aurait agi », « était censée agir », hm ?

En anglais

L’internet en anglais est des dizaines de fois plus étendu que celui dans la pauvre langue française. Voyons ce qu’il peut nous offrir sur le sujet :

Pour commencer on rencontre les inévitables forums avec petit chef qui vient immédiatement, sorti d’on ne sait où, expliquer que TOUT dans l’univers, doit respecter la loi d’Einstein :

http://www.thenakedscientists.com/HTML/questions/question/1000305/

http://physics.stackexchange.com/questions/5456/the-speed-of-gravity

http://physics.stackexchange.com/questions/12736/does-gravity-spread-instantly

https://www.physicsforums.com/threads/is-gravity-instant.499985/

Le premier troll commence à expliquer que cette vitesse égale à c est « évidemment » un peu difficile à tester (et là on commence à comprendre qu’il confond bien sûr, avec les ondes gravitationnelles, etc.)

On retrouve aussi les « gurus » sur ces soi-disant forums où les rôles sont en fait bien définis : toi tu poses les questions, moi, Ethan Siefel, je réponds : https://medium.com/starts-with-a-bang/what-is-the-speed-of-gravity-8ada2eb08430

Ethan Siefel confond complètement et furieusement, évidemment, la gravitation et les supposées « ondes gravitationnelles ».

Les pages wikipedia colportent les mêmes histoires, comme ici :

https://en.wikipedia.org/wiki/Gravity#General_relativity

Grossièrement, Newton avait faux, mais la relativité générale à corrigé tout cela. Suit une liste des « vérifications », des prédictions réalisées de la RG. Pas de liste des « problèmes » de la RG pour décrire l’univers.

ou là : https://en.wikipedia.org/wiki/Speed_of_gravity#Static_fields

Le rédacteur de ce paragraphe se démène pour prouver que la gravitation NE PEUT être instantanée, bien qu’en pratique IL FAILLE qu’elle le paraisse, à cause de la symétrie nécessaire des observations dans différents repères ! Il ne semble pas se rendre compte que sa modification instantanée des champs prétendument « statiques » lors d’un déplacement de l’émetteur ou du récepteur gravitationnels signifie une connaissance instantanée de la direction et de la distance de la source, donc une propagation INSTANTANÉE d’une information, alors qu’il essaie à toute force de démontrer que RIEN ne se propage.

Mais il y a aussi sur internet quelques fausses notes :

http://www2.lbl.gov/Science-Articles/Archive/Phys-speed-of-gravity.html

Ici Stuart Samuel, un physicien qui travaille dans une université (euh juste à Berkeley, en Californie, l’équivalent de Romorantin en France), et qui croit d’ailleurs qu’ Einstein a prédit que la gravitation se propageait à la vitesse de la lumière (? Einstein a prédit ça ? en fait non, voir ma conclusion ) pense que c’est peut-être vrai, mais cela n’a jamais été prouvé.

Au niveau de l’expérience dans laquelle la lumière d’un quasar passe à côté de Jupiter (et on étudie comment se propage la gravité de Jupiter en étudiant son action sur la lumière du quasar) il pense que les calculs (de ceux qui ont crié victoire) : https://www.newscientist.com/article/dn3232-first-speed-of-gravity-measurement-revealed#.VZq-yxuqqko

(le New Scientist est l’équivalent britannique de La Recherche en France) sont faux et que les vrais calculs donnent des différences à trouver qui ne sont pas mesurable avec les instruments actuels….

Les autres scientifiques, comme un « expert » nommé Lawrence Krauss de la Case Western Reserve University de Cleveland, Ohio par exemple ont dit plutôt : « ah ben ouais, c’est normal, Einstein l’avait dit, on le savait déjà donc, pas la peine de mesurer en fait, bon c’est pas le tout ça, il est vendredi soir, j’ai pétanque moi » (traduit de l’américain par votre serviteur, voir la fin de l’article du New Scientist).

La page http://math.ucr.edu/home/baez/physics/Relativity/GR/grav_speed.html a pour titre :

Does Gravity Travel at the Speed of Light?

donc je suis obligé d’en parler ici. C’est un site qui pose (et répond) à des questions de physique..

En bref, l’auteur pense que la gravité était instantanée dans les modèles pré-relativistes (c’est clair) mais qu’elle ne peut plus l’être depuis. Toutefois, après quelques contorsions, il arrive à l’idée que le « délai de propagation est annulé par le fait que la force s’exerce de façon non centrale » (c.a.d non dirigée exactement vers la source de la gravité) et qu’au final la « gravitation relativiste reproduit presque parfaitement le modèle newtonien ».

La fin de l’article aboutit à des considérations sur les ondes gravitationnelles, qui ne sont pas détectées, mais presque, et quand on les trouvera, elle iront à la vitesse de la lumière, donc la gravité aussi, madame. Ceci est toutefois précédé par une pseudo démonstration (en un seul début de phrase) comme quoi un champ a nécessairement la même vitesse de propagation que les ondes qu’il supporte (si j’ai bien compris).

Cet auteur ne semble pas conscient qu’une vitesse finie de propagation du champ gravitique pourrait avoir DEUX effets distincts, le retard dû au délai ET aberration gravitaire.

Voici un autre auteur qui est spécialiste de cette distinction, pour la lumière en tout cas :

http://www.mathpages.com/home/kmath690/kmath690.htm

Son titre aussi attaque le problème de front : Laplace on the Speed of Gravity !

Très long article, mais si vous avez le courage de le lire (en assimilant toutes les formules, bien entendu) vous en serez récompensés ! C’est une grande récapitulation de l’histoire scientifique de la gravitation et des questions sur sa vitesse.

Deux types de gravitation sont évoquées, une tirante (celle de Laplace, qui pensait que des particules étaient émises par la source et captées par le sujet gravitant, une sorte de pré-graviton quoi !) et une poussante, où la gravitation est due à un effet de masquage/ombrage dans une mer de particule qui poussent tout le temps sur tous les corps.

Malheureusement, l’expérience choisie à l’époque par Laplace pour tester le retard de la gravitation (incluant à la fois retardation et aberration) était faux. En effet, il a été prouvé depuis que le ralentissement absolu de la période lunaire (et le ralentissement de la révolution terrestre) sont dus aux marées, une accélération de la première étant observée en pratique. Il aurait fallu étudier plutôt la variation de la durée l’année (mais comment ? les hommes préhistoriques n’avaient pas d’horloges atomiques paraît-il).

L’article offre lui aussi l’idée (malheureusement très embrouillée car assise sur la critique de trois articles) que la propagation de la gravité est bien finie, d’où une aberration (nom que l’auteur donne à la somme de la retardation ET de l’effet Bradley, qui est l’aberration classique), mais que des effets relativistes dus à la vitesse l’annulent « presque exactement ». Toutefois, chez un auteur très prolixe en calculs, en dates, en schémas et en citations, aucune référence n’est donnée sur l’origine, le calcul, la représentation etc. de cette « annulation presque exacte » contrairement à ce qu’il fait longuement :

retardation de l'action à distance

dans le cas de l’ancienne théorie de Lesage (http//www.mathpages.com/home/kmath131/kmath131.htm). Dans le cadre d’une gravité due à un champ, donc non corpusculaire, il est d’ailleurs probable qu’il n’y aurait pas « d’effet Bradley » comme il le nomme, mais il faudrait toujours traiter la « retardation », soit le retard d’information sur la position des corps en jeu.

En somme, l’auteur est très fort en calculs, mais il ne semble pas se rendre compte qu’une annulation « presque » exacte seulement produirait tout de même un effet mesurable, surtout sur les planètes lointaines. Et pourtant, c’est bien le même auteur, qui écrit (à la première personne apparemment) dans l’autre article cité plus haut :  » In order for the orbits of the planets around the Sun to have persisted for hundreds of millions of years, the ratio of tangential to radial acceleration can be no greater than about 10¨-10, and we’ve already noted that the fraction k of ultra-mundane particles absorbed by the Sun can be no greater than 10¨-8, so the above relation implies that the speed of the ultra-mundane particles must be at least (6)10¨18 times the speed v of the planet.  The planet Mercury has an orbital speed of about (4.8)10¨4 m/sec, so the value of vg must be at least (2.9)10¨23 m/sec, which is about 10¨15 times the speed of light. Henri Poincare used more stringent limits on the shielding and acceleration ratios to conclude that vg must be about 10¨17 times the speed of light« .

CONCLUSION

Le sujet est vaste, très complexe et il est difficile de s’y retrouver si on se lance sans points de repères (ce que semblent faire de nombreux articles). Aussi ma méthode sera de prendre des points précis de réflexion et d’analyser comment la question se pose, quelles erreurs ou confusions peuvent être faites à ce sujet.

La gravitation telle qu’elle fut expliquée par Newton

La gravitation de Newton est en réalité composée de deux sous-parties mal intégrées au sein de la même théorie (ce qui est normal compte tenu de l’époque) :

a) l’action à distance (d’un corps sur un autre); cette partie a été mathématisée par Newton dans sa célèbre formule; la formule n’est pas vectorisée et ne contient pas non plus de facteur temps. Le calcul ne produit que la moitié du résultat observé dans la nature. Newton s’est basé sur un calcul d’action en inverse du carré de la distance qui préexistait dans le monde mathématique de son époque.

b) l’instantanéité de cette action. Elle n’est pas mathématisée autrement que par la géométrie et cette vision géométrique représente de fait les corps (possiblement) mouvants comme s’ils étaient immobiles (la force agissant sur une masse pointe sur le centre de gravité de l’autre masse).

Au cours de la rédaction de cette page, j’ai donc rencontré de nombreux auteurs qui écrivent que Newton lui-même était très étonné par l’action instantanée à distance. Je crois qu’ils ont mal lu Newton, ou plus probablement qu’ils injectent dans sa pensée une préoccupation contemporaine. Voici le texte de Newton qui est d’ailleurs souvent cité, mais sans que les citateurs l’aient lu attentivement, à mon avis :

« It is inconceivable, that inanimate brute matter should, without the mediation of something else, which is not material, operate upon and affect other matter without mutual contact … That gravity should be innate, inherent, and essential to matter, so that one body may act upon another at a distance, through a vacuum, without the mediation of anything else, by and through which their action and force may be conveyed from one to another, is to me so great an absurdity, that I believe no man who has in philosophical matters a competent faculty of thinking, can ever fall into it. Gravity must be caused by an agent, acting constantly according to certain laws; but whether this agent be material or immaterial, I have left to the consideration of my readers.  [Sir Isaac Newton,Third letter to Bentley, 25 Feb 1693. Quoted in The Works of Richard Bentley, D.D. (1838), Vol. 3, 212-3.]

Une lecture objective montre qu’il n’est aucunement fait mention dans cette réflexion post-partem de l’instantanéité, mais seulement de l’action à distance. Newton est gêné que l’action de la gravité puisse se faire sentir à travers le vide, mais nullement qu’elle puisse se faire sentir de façon instantanée. À cela, une bonne raison : la vitesse finie de la lumière (Cassini 1675) est seulement en train d’être banalisée (150 ans, c’est récent à l’époque) ! À l’époque tout le monde est encore partiellement OK avec une lumière plus ou moins instantanée, alors si la lumière l’est, la gravité peut sans souci particulier l’être aussi.

Par ailleurs j’ajouterai que cette remarque révèle un paradoxe dans la pensée de Newton (non signalé ailleurs) : alors même qu’il est crédité d’avoir compris le premier, comme je l’ai écrit plus haut déjà, que la même loi de la nature s’appliquait à la pomme et à la lune, il est clair que son étonnement par rapport à l’action « à distance » ne s’applique en fait qu’à la LUNE ! La mention du vide est significative. Il trouve normal que la terre attire la pomme car la pomme se trouve en quelque sorte « à son contact », proche, tandis que la lune est séparée de la terre par une grande distance,et cette distance est remplie de « vide ».. En somme il a posé une loi universelle, mais il la trouve absurde. En son for intérieur, il n’y croit pas à sa loi. Et ce qui est absurde pour lui dans cette loi, c’est le cœur même de son universalité !

La « correction »  de la gravité  newtonienne par Einstein

La relativité restreinte (special relativity en anglais) vient en premier. Elle ne parle pas de la gravitation, seulement de la vitesse de la lumière

Ensuite vient la relativité générale (general relativity), qui intègre la première relativité ET des remarques et prédictions sur la gravité. Du coup les exégètes en font une « théorie de la gravitation ».

C’est difficilement soutenable. En effet comme on l’a vu, la théorie de Newton se compose de deux volets, l’action à distance et l’instantanéité. La relativité générale s’est clairement attaqué à la première partie, l’action à distance.

Le concept d’action à distance est incompatible avec la relativité car en raccourci, une action a besoin d’un milieu ou d’un vecteur pour se transmettre or un des piliers de la théorie de la relativité a été de détruire la notion d’éther, le milieu qui aurait pu transmettre, justement la gravitation.

Pour contourner ce problème, Einstein a posé l’idée que la gravitation était le résultat d’une déformation de l’espace-temps (on en reparlera). Dans ce contexte, le poids n’est plus une une force, mais le manifestation d’une tendance à changer de trajectoire dans l’espace temps. J’avais lu dans le temps que dans un espace temps déformé, certains endroits de l’espace pouvaient contenir plus de matière, d’où la tendance de la matière à s’y précipiter. Je n’ai pas retrouvé cette explication, seulement celle du « trampoline » que je discuterai plus tard.

La relativité générale ne s’est jamais attaquée au second volet de la gravitation de Newton, l’instantanéité ET POUR CAUSE :

L’instantanéité de l’effet gravitationnel RESTE EN VIGUEUR et Einstein le savait parfaitement. Cette instantanéité peut être démontrée facilement à plusieurs échelles :

A l’échelle du système solaire, la trajectoire des planètes (non relativistes) et de leurs satellites continue d’être expliquée parfaitement par les lois de Kepler/Newton. Tout angle, même très petit entre la direction du vraie du soleil et la direction de son attraction aurait une répercussion énorme sur les orbites, surtout à long terme, or la stabilité de ces orbites est prouvée par l’astronomie historique.

Au niveau des petites poussières en orbite autour du soleil, l’effet Lense-Thirring n’existe que parce que la pression de radiation a une aberration, elle, MAIS PAS LEUR VECTEUR-POIDS. Si une pareille aberration touchait le poids des poussières en question, l’effet serait différent (conformément au principe de schizophrénie avancé au début du présent article, le poids n’est pas représenté sur les schémas de https://fr.wikipedia.org/wiki/Effet_Lense-Thirring)

Au niveau galactique et supra-galactique, et quelle que soit la théorie sur leur formation/évolution, l’existence même des galaxies en tant qu’objets ayant une cohésion gravitationnelle ne peut avoir lieu que par une transmission extrêmement rapide de la gravité d’un bout à l’autre de ces choses qui font jusqu’à 1 million d’années lumière de diamètre !

Les galaxies sont des objets essentiellement symétriques. Or elles ont un sens de rotation (si on s’abstrait des petits mouvements propres des étoiles). Si la gravité avait une vitesse finie, les étoiles situées à une certaine distance du « centre » de la galaxie, mettons 300 000 al ne pourraient pas tourner autour du même centre que les étoiles proches de ce centre car entre temps le centre se serait déplacé pendant 300 000 ans et surtout ne devrait- on pas détecter une asymétrie entre les étoiles qui vont vers « l’avant » et celle qui vont vers « l’arrière » du mouvement de cette galaxie ?

Prédiction supposée sur la vitesse de la gravitation par la relativité générale

Einstein n’a jamais affirmé ni prédit que la vitesse de la gravité serait trouvée comme étant égale à c !

Einstein a prédit qu’il existait des « ONDES GRAVITATIONNELLES » qui auraient c pour vitesse. Les ondes gravitationnelles étant conçues comme des phénomènes relativistes, donc rares, et interagissant très peu avec la matière.

La gravité elle même n’est pas rare, c’est le phénomène dominant de l’univers et elle interagit de façon évidente avec la matière sauf à l’échelle microscopique et en dessous.

On l’a vu, cette croyance est due au fait que de nombreux trolls, blogueurs, et ce qui est plus inquiétant, certains universitaires, CONFONDENT gravitation d’une part et ondes gravitationnelles d’autre part !

Par ailleurs, Einstein n’ignorait surement pas, contrairement à eux, que Laplace a prouvé de manière définitive que la vitesse minimum de la gravité (à partir de mesures dans le système solaire) devait être au minimum 7 millions de fois supérieur à c. !

cf. https://en.wikipedia.org/wiki/Speed_of_gravity#Laplace et même si l’auteur se donne beaucoup de mal pour arriver à l’idée que cette idée est « incorrecte » il finit lui-même par écrire « As is now known, it may be considered to be infinite in the limit of straight-line motion, since as a static influence, it is instantaneous at distance, when seen by observers at constant transverse velocity. For orbits in which velocity (direction of speed) changes slowly, it is almost infinite. » Autrement dit l’idée est « incorrecte » mais toujours vraie.

Enfin, dernière question, il parait que des étudiants la posent (j’espère qu’on recale ces étudiants immédiatement) : si la gravitation se propageait à la vitesse de la lumière, comment ferait-elle pour « sortir » des trous noirs ? On sait en effet que justement, par définition, il y a trou noir quand la lumière ne peut plus en sortir (ça doit pas être noir du tout à l’intérieur … quelle idée puissante de la physique je viens d’émettre !) mais qu’ils sont toujours détectables par leurs effets gravitationnels (et pour cause car c’est comme ça qu’on les détecte justement). Donc si la lumière ne peut pas sortir du trou noir « parce qu’elle ne va pas assez vite » mais que la gravité sort du trou noir, qu’en concluez vous ?

Comparaison gravité accélération

Un des grands faits d’armes de la relativité générale est la mise en équivalence de la gravitation et de l’accélération.

Dans une expérience de pensée classique, un observateur dans une enceinte sans fenêtres ne peut pas savoir s’il est à la surface d’une planète et subit son attraction ou bien s’il est au contraire dans un vaisseau spatial en train d’accélérer. Certes cette ‘expérience’ a permis à Einstein de prévoir la courbure de la lumière dans un champ gravitationnel (confirmée : passage d’une lumière d’origine distante près du soleil, lentilles gravitationnelles). Quelques prédictions intéressantes ne font pas de la relativité générale une vraie théorie de la gravitation. Elle n’explique pas réellement comment la matière « produit » de la gravité et encore moins comment elle se propage apparemment instantanément.

Elevator_gravity2

Mais revenons sur l’expérience de pensée, reproduite un peu partout : elle est fausse !

  • l’accélération est la même pour toutes les parties de la fusée et des choses/personnes qui se trouvent à l’intérieur. En revanche, à la surface d’une planète, la gravité a toujours un gradient. Muni d’un instrument assez sensible, l’expérimentateur enfermé pourrait donc distinguer immédiatement la gravité artificielle (égale partout) d’une véritable attraction gravitationnelle (plus forte à ses pieds qu’à sa tête). La gravité n’est constante en force et en direction que dans les exercices de physique, pas dans la réalité de la gravitation dans notre univers ! Dans une chute libre dans un vrai champ de gravité, contrairement à dans la fusée d’Einstein, deux objets en chute libre vont immédiatement commencer à se rapprocher (même potentiel de départ) ou s’éloigner (potentiels différents au départ).
  •  

La disparition du Soleil

De nombreux astronomes en culotte courte se posent des questions sur la gravité en terme catastrophiques. Incapable de comprendre que l’effet de la gravité peut très bien être étudié en continu grâce au mouvement des astres, il leur faut toujours une énorme catastrophe pour savoir ce qui se passera après. En général ils aiment bien postuler la DISPARITION DU SOLEIL comme on peut s’en rendre compte en épluchant de nombreux blogs, toutes langues confondues. C’est probablement et tout simplement une habitude, prise lors de la même disparition du soleil, souvent envisagée pour savoir ce qu’il adviendrait de notre éclairage sur Terre.

On en a déjà vu des exemples, avec représentation des ondes gravitationnelles dans le même repère que la gravité. En voici un autre : https://www.youtube.com/watch?v=0rocNtnD-yI

Le modèle pédagogique du trampoline

Le grand truc pour expliquer la courbure de l’espace-temps (soi-disant) est le trampoline sur lequel on place des boules. Les boules dépriment la surface du tissu (et les lecteurs de la démonstration) et permettent ainsi d’imaginer en 2 D ce qui se passe supposément en 4D.

  • Première remarque (critique qui n’est pas nouvelle, je l’ai lue à plusieurs reprises) : cette représentation destinée à « expliquer » la gravitation ne marche en fait que GRACE à la gravitation. Les boules creusent la toile d’une part, et d’autre part se mettent à rouler ou modifient leur trajectoires sur cette toile uniquement à cause de la gravité terrestre, de leur poids. En somme cette explication a tout l’air d’une supercherie intellectuelle puisqu’elle ne peut expliquer la gravitation, qu’à partir d’un de ses effets (le potentiel lié à l’altitude).

Mes remarques

  • Bien que les relativistes insistent beaucoup sur le fait que c’est l’espace-TEMPS qui est courbé, la modélisation par le trampoline apparaît fortement biaisée vers un espace (en 2D) qui est courbé dans la troisième dimension, toujours d’espace, où est le temps dans tout ça ?
  • Un gros objet est capable de creuser le trampoline et d’attirer ainsi tout objet plus petit qui passe à côté mais la réciprocité de la gravitation est mal modélisée car le petit objet ne fait pas rouler le gros vers lui (on verra que ce défaut est corrigé dans les modèles animés ci-dessous)
  • La personne qui a inventé ce modèle devrait changer de matelas s’il se retrouve toujours collé à sa (son) partenaire au milieu de la nuit (et du lit).

Quelques représentations :

BangIllus5-590x348

Comme souligné plus haut un grave défaut du « modèle du trampoline » est qu’il tend à présenter la gravité comme l’interaction d’un « gros » objet immobile, qui « produit » le champ de gravité avec un « petit » objet mobile, qui lui la subit. Cela entraîne l’idée que le champ étant « déjà là » au moment où on pénètre à l’intérieur de son influence, il n’a pas besoin de se propager, ce qui explique (pour ceux qui croient à cette préexistence du champ) son apparente instantanéité…

Spacetime_curvature

Ces représentations ont du mal également à représenter la portée infinie de la gravitation. La trame de l’espace temps semble tout à fait inaffectée dès que l’on est à un une certaine distance de la grosse masse.

Dans une représentation animée, on observe que les défauts mentionnés plus haut ont été corrigés :

Trois corps sont représentés : la Lune tourne autour de la Terre (en rotation !) mais les deux tournent ensemble autour du Soleil ! Par ailleurs le trampoline ne semble pas immobile par rapport au Soleil, qui paraît lui même en train de se déplacer dans l’espace. Quelle amélioration, même si le modèle reste essentiellement spatial et non spatio-temporel !

Malheureusement le concepteur de ces magnifiques animations croit qu’il est en train de faire des vraies expériences de physique et il en profite pour nous expliquer comment les planètes vont s’échapper si le SOLEIL venait à disparaître, pouf :

PAS COMME CA (vilain Newton) :

ExpGrav-01.gif

Gravité instantanée = PAS BON car Einstein a dit que etc…

Mais comme ça :

ExpGrav-02.gif

(je crois que l’animateur s’est trompé, dans la vue « d’oiseau », la planète s’enfuit avant que l’obscurité ne la touche ??? Évènements qui semblent simultanées dans la vue de profil ???)

Arrivé là, pourquoi se gêner ? Si le trampoline peut représenter la gravité, il peut aussi représenter les ondes gravitationnelles, non ?

ExpGrav-04.gif

Le soleil disparaît brusquement, la cessation de sa gravité et l’onde gravitationnelle se propagent simultanément à la vitesse de la lumière. La planète oublie toutefois de s’éteindre, m’enfin ?

Une autre représentation trampolinesque des ondes gravitationnelles :

Une telle perturbation déformerait la géométrie de l’espace (et des choses contenues), d’accord mais c’est une très grave erreur mathématique de représenter da la même manière la gravitation ET les ondes gravitationnelles, qui n’ont rien à voir. L’axe z (vertical) qui sert (modèle du trampoline) à représenter l’intensité du champ de gravité (plus exactement le vecteur pente comme dérivée du vecteur gravité) ne peut pas servir sur le même schéma à représenter autre chose, on ne sait pas bien quoi d’ailleurs, la dilatation latérale ?

Vidéos :

Ici un professeur a réellement construit un trampoline avec un tissu caoutchouteux à souhait :

 

Là une vidéo sur les ondes gravitationnelle, remarquez l’utilisation du trampoline qui risque de mélanger l’esprit des visionneurs :

Dernières nouvelles du LIGO :

Allez vous reprendrez bien un petit coup de trampoline ? :

http://www.lefigaro.fr/sciences/2016/02/11/01008-20160211ARTFIG00365-on-a-vu-les-ondes-gravitationnelles-prevues-par-einstein.php

INFa64a86fe-d0c7-11e5-8a31-fe797bd7bb40-805x453

Les ondes gravitationnelles sont elles en fait ‘gravitationnelles’ ?

Comme on l’a vu par de nombreux exemples, les ondes gravitationnelles font régulièrement l’objet d une confusion avec la gravitation elle-même.

Une raison primaire de cette confusion réside en fait dans leur nom, qui est mal choisi.
 
D’après la théorie même qui les prévoit, ce sont des ondes d’ORIGINE gravitationnelle, non des ondes de GRAVITATION.
 
Elles n’ont aucun effet gravitationnel, c’est à dire qu’elle n’agissent pas sur le champ vectoriel de gravitation mais sur la texture de l’espace rempli ou non de matière. Sous leur influence les objets et l’espace vide voient leurs dimensions vibrer.
 
Il conviendrait donc de les appeler ‘ondes spatio-temporelles ».
 
Dans une présentation de Luminet « Les trous noirs: Les trous noirs physique et astrophysique  » (2015), les ondes gravitationnelles du titre se transforment subrepticement en « ondes de gravitation » dans le dessin, voilà qui n’est pas fait pour mettre fin à la confusion :
 
ondes de gravitation
On verra à la fin de cet article que Luminet est coutumier de cette confusion ondes G/gravitation…
 

 

Champs de gravité = champs scalaire ?

On lit parfois que le champ de gravité serait un champ scalaire, c’est à dire qu’en tout point de l’espace, un seul paramètre numérique lui serait associé. Un champ scalaire bien connu est le champ de pression à l’intérieur d’un gaz/fluide/solide. En chaque point la valeur numérique de la pression n’a pas de direction. Sur n’importe quelle objet étendu se trouvant autour de ce point, elle s’exercera quelle que soit direction de sa surface. Il est certain que c’est faux dans le cas de la gravité qui, en chaque point, a un vecteur associé. Ce vecteur, comme tout vecteur, porte en lui une valeur numérique, son module, mais aussi un axe et un signe. Tout objet (masse en l’occurrence) placé en ce point recevra une accélération dirigée selon ce vecteur. Le champ de gravité n’est pas scalaire. (je n’ai pas retrouvé cette erreur toutefois, aurais-je rêvé ?)

Correction des aberrations de la gravité par des effets dus à la vitesse

On a vu que certains auteurs pensent que la gravité se propage à la vitesse de la lumière « parce que c’est comme ça » sans tenir compte de la réalité astronomique. D’autres ont développé une théorie du champ statique qui n’a pas besoin de se propager (parce qu’il est déjà là)… La troisième façon de « lutter » contre l’instantanéité de la pesanteur est de se référer aux « effets dus à la vitesse » comme le fait Kevin Brown dans son livre Reflexions on Relativity (pages citées plus haut sur son site mathpages, du moins on suppose que c’est de lui !).

J’ai fini par localiser l’origine de cette théorie, il s’agit d’un article de S. Carlip du département de physique de l’université de Californie.

http://arxiv.org/pdf/gr-qc/9909087v2.pdf

Je n’ai pas le niveau mathématique pour critiquer les équations qu’il aligne. Il me semble pourtant que la fin de l’article consiste bien en une redescente au niveau philosophique où, tel le blogueur moyen ou le journaliste de La Recherche, Carlip finit par des arguments embrouillés qui expliquent que, bien que « the observed absence of aberration is consistent with instantaneous propagation » cela est « also consistent with the speed-of-light propagation predicted by general relativity. » (on se demande bien comment et quelles observations sont aussi bien compatibles avec une vitesse infinie et une vitesse finie ?).

Les simulations

Les ordinateurs permettent de simuler à peu près n’importe quoi et l’internet de vulgariser les travaux extrêmement facilement ! On peut donc trouver :

Des programmes amateurs, comme ici

http://www.nowykurier.com/toys/gravity/gravity.html (programme en 2D, soit un univers plat, dans lequel est modalisé la formation d’un système solaire)

le résultat de simulations réalisées par les astrophysiciens, comme dans cette vidéo

Mais si vous n’avez pas de super-calculateur à la maison, vous pourrez quand même jouer en 3D, avec les simulations du concours lançé par la Khan Academy.

Voici le programme gagnant : https://www.khanacademy.org/cosmology-and-astronomy/challenge-modeling-accretion-disks/1180451277

Ce jouet est extraordinaire mais en plus, vous pouvez modifier le code et relancer la simulation pour voir la différence !

Si vous voulez voir les résultats obtenus par d’autres programmeurs (certains sont très proches (?) mais d’autres sont partis sur des bases différentes, allez à la page : https://www.khanacademy.org/science/cosmology-and-astronomy/stellar-life-topic/stellar-life-death-tutorial/p/challenge-modeling-accretion-disks

puis cliquez sur l’onglet spin-offs (au pluriel, pas sur le bouton spin-off au singulier !!!)

Ces simulations « amateurs » semblent plutôt modéliser la formation d’un système solaire à partir d’un nuage de rochers que la formation d’une galaxie, mais bon.

Comme vous l’avez deviné, elles utilisent toutes une gravitation instantanée, c’est à dire que l’accélération de chaque objet est calculée en fonction de la position (et de la masse évidemment) de tous les autres au moment du calcul…

(Je vais dès que possible (3/2016) rédiger une demande pour construire une variante où la force (donc l’accélération) imprimée par chaque objet serait retardée donc calculée en intensité et surtout en direction par rapport à une position d’autant reculée dans le temps qu’il serait éloigné. Cela ne semble pas impossible puisque les distances entre chaque objets sont calculées à partir des positions. Par contre il se peut que les anciennes positions ne soient pas stockées donc accessibles. Une solution élégante serait de tracer la trajectoire d’un objet (comme dans le programme « nowykurier » quand on clique sur Paths. Retrouver la position et la direction d’un attracteur tel que « vu » par un attracté revient alors à faire grandir un globe autour de l’objet attiré à la vitesse choisie pour la gravitation dans cet univers, tandis qu’un point remonte la trajectoire en arrière à la même vitesse. Au moment où le point coïncide avec la surface de la sphère, on a l’ancienne position de l’attracteur …)

Cette demande n’a jamais reçu de réponse … mais incroyablement, quelqu’un s’est amusé à construire cette simulation. Je la donnerai dans mon 4ème article :

 

La gravité est-elle instantanée ou le café seulement 4 ?

 

En guise d’épilogue : la gravitation sera-t-elle quantifiée, toutes les forces unifiées ?

La théorie de la relativité procure une compréhension presque totale de l’univers à l’échelle cosmique, enfin soi-disant car elle ne prévoit pas la matière noire, encore moins l’énergie noire.

La théorie quantique procure une compréhension presque totale de l’univers à l’échelle atomique, enfin soi-disant car elle ne prévoit pas non plus ces négritudes…

Le grand rêve de la physique serait d’unifier ces deux théories ou bien d’accéder à une théorie qui puisse les englober toutes les deux.

Pour cela une nécessité serait avant tout de quantifier la gravitation, ce qui revient aussi à isoler une particule élémentaire qui en serait le vecteur, soit un « graviton » qui serait à la gravité ce que le photon est à la lumière. Inutile de dire que ce graviton se déplacerait à la vitesse de la lumière etc.

Cette  unification est présentée par de nombreux auteurs, qui ne craignent pas de prendre leurs désirs pour des réalités, comme inéluctable. Je vais expliquer ici sans aucun calcul pourquoi elle n’arrivera jamais et pourquoi le graviton ne sera jamais découvert !

Dans les interactions quantiques, quand une particule réagit avec une cible, cette particule disparaît ou est perturbée d’une autre façon. Par exemple si un photon est absorbé (photosynthèse, effet photo-électrique), par définition, il ne peut pas continuer son chemin.

Or la gravitation a une caractéristique qui n’est pas trop abordée dans les livres auxquels j’accède et qui découle pourtant tout simplement du fait qu’elle est comprise comme un champ : le champ agit sans être absorbé.

Pour prendre des exemples concrets, la gravité que nous ressentons à la surface de la terre est la somme des effets de l’attraction de tous ces atomes sur ceux de notre corps, sachant que la plupart de ces effets se produisent à travers des centaines et des milliers de kilomètres de matière, la terre elle-même !

De même lorsque la lune occulte le soleil, la terre continue tout de même sur l’orbite qui est la sienne car la gravité du soleil « passe à travers la lune ». Aucune particule quantique ne permet cet effet AMHA.

Supplément spécial de dernière minute : Luminet est-il si lumineux ?

Dans son blog http://blogs.futura-sciences.com/luminet, Jean-Pierre Luminet commence en février 2016 un nouvel article : « LA « LUMIÈRE » GRAVITATIONNELLE » (le texte  de la page au 30/10/2016 : la-lumiere-gravitationnelle-1-4-principes-de-base-par-jean-pierre-luminet)

Je procéderai en extrayant quelques lignes de ce blog et en les commentant :

« Je voudrais poser une question à monsieur Einstein, à savoir, à quelle vitesse l’action de la gravitation se propage-t-elle dans votre théorie ?
Max Born, 1913″

Cette phrase est mise en exergue, il est clair que l’article va parler de la propagation de l’effet de la gravitation, même si le titre était un peu troublant (l’adjectif « gravitationnel(le) » se rapportant plus souvent aux « ondes G » qu’à la gravitation elle-même)…

Continuons à lire :

« Dans la théorie de Newton, la gravitation est une force agissant instantanément entre les corps massifs. Cette idée était inadmissible aux yeux de nombreux physiciens, Newton compris,« 

Comme on l’a vu, la gravitation dans la théorie de Newton est plutôt un couple de forces qui agissent réciproquement et Luminet répète lui-aussi cette fable d’un Newton qui trouverait inadmissible sa PROPRE théorie ….

Et comme je l’ai montré plus haut, il ressort de sa lettre que ce que Newton trouvait difficile à admettre, c’est l’action à distance, pas l’instantanéité, qui d’ailleurs ne se présente pas vraiment à l’esprit,  dans sa formulation non vectorielle, pré-scientifique.

« un siècle plus tard Laplace proposait une modification de la théorie dans laquelle l’interaction gravitationnelle se propageait à vitesse finie. »

C’est une déformation de l’histoire : en réalité Laplace, qui voulait trouver par une expérience la vitesse (qu’il supposait finie, comme celle de la lumière) de la gravité, a été obligé de concéder à cette dernière une vitesse très supérieure à celle de la lumière, en pratique une borne inférieure à cette vitesse de la gravité, qu’il ne put mesurer !

« une question à laquelle personne ne savait répondre : lorsqu’un corps massif est violemment perturbé, le champ gravitationnel qu’il engendre doit s’ajuster de proche en proche » (doit-je colorier ça en vert ou en rouge à votre avis ?)

Le champ gravitationnel doit s’ajuster, c’est sûr, mais même si le corps n’est PAS violemment perturbé (à quoi pensez-vous au juste M. Luminet en écrivant cela ?) et « de proche en proche« , oui certes, en supposant qu’il ait une vitesse, ce réajustement…

« La théorie de la relativité générale d’Einstein permet d’organiser en un schéma cohérent les intuitions sur la propagation de la gravitation.« 

Nous sommes contents de l’apprendre, Luminet a dû trouver une oeuvre ignorée d’Einstein (je n’ai trouvé aucune citation d’Einstein faisant des prévisions sur la propagation de la gravitation, merci de me détromper si vous en connaissez une)

« Einstein s’était demandé si une masse en mouvement accéléré pouvait rayonner des ondes de gravité, de la même façon qu’une charge électrique en mouvement accéléré rayonne des ondes électromagnétiques. »

Toutes, ou aucune, masse ne sont « en mouvement » et que sont les « ondes de gravité« , vous voulez dire les ondes gravitationnelles ? Mais vous parliez dans la phrase précédente de la GRAVITATION ?

‘Une autre complication vient de ce que le graviton, l’hypothétique particule médiatrice de l’onde gravitationnelle, « 

Ah bon, depuis quand ? Il est écrit à peu près partout que le graviton est le médiateur de la GRAVITE : https://fr.wikipedia.org/wiki/Graviton

« Si l’on place à 1 centimètre l’un de l’autre deux protons – qui, ayant une masse et une charge électrique, sont à la fois soumis à une interaction gravitationnelle et à une interaction électromagnétique –, la force gravitationnelle qui les attire est 1037 fois plus faible que la force électrostatique qui les fait se repousser. De là vient l’obstacle majeur à la détection des ondes gravitationnelles « 

Donc la force gravitationnelle, c’est la même chose que les ondes gravitationnelles ?

Quel catastrophique GLOUBIBOULGA !

********************************************

Mise à jour de janvier 2017

 
Au cours de mes récentes lectures, j’ai trouvé ces autres sources, toutes entachées de la même confusion :
 
 
Dans cette vidéo pédagogique l’impétrant qualifie les ondes gravitationnelles d’ondes de déformation (je crois) et immédiatement dans la foulée, d’ondes de COURBURE (de l’E.T.)
 
 
 
dans laquelle le paragraphe

Quantum gravity, wave-particle aspects, and graviton

est entièrement à propos de la gravité, plus du tout des ondes gravitationnelles.
 
– Enfin la thèse de
 
 
Cherchons y les mentions de « ondes gravitationelles » :
 
Les 5 premières occurences parlent bien des OG, mais à la
 
6ème citation  « Nous réhabilitons toute une classe de modèles généralisant les théories tenseur-scalaires et qui avait été, à notre avis, trop vite abandonnée. Ces modèles nécessitent en effet de considérer des ondes gravitationnelles supraluminiques pour reproduire la phénoménologie MONDienne, et en particulier la déflexion de la lumière produite par les objets astrophysiques dominés par la matière noire »
 
on a clairement zappé sur la gravité ?
 
et puis page 21 trois citations qui parlent nettement parler de la gravité, pas des ondes gravitationnelles.
 
à partir de la page 29, retour au ondes gravitationnelles
 
page 120 et suivante, retour à la gravitation (les ondes gravitationnelles ne peuvent dévier la lumière)
 
dans les parties en anglais, l’auteur utilise clairement « gravitationnal waves » comme le vecteur de la gravité (« We define superluminal behavior as going faster than gravitons [i.e., gravitational waves] » dans tout le début
 
mais tout à coup à la page 66 :
 
« 6.2 Introduction The detection and analysis of gravitational waves on Earth will open a new domain of high energy astrophysics »
 
il a évidemment basculé sur les ondes d’origine gravitationnelle…
 
et à la page 125 il a de nouveau rebasculé sur la gravité : « However, the opposite phenomenon occurs in TeVeS : photons (and high-energy matter particles) propagate faster than gravitons. References [180, 74] actually proved that the observation of high-energy cosmic rays imposes tight constraints on such a behavior, because these rays should have lost their energy by Cerenkov radiation of gravitational waves. »
 
(je m’arrête avant la fin de l’oeuvre, continuez si vous voulez)
 
Maintenant il est très possible que cette thèse de haut niveau, dont je ne peux suivre tous les raisonnemnents logiques et encore moins tous les calculs, soit pleine de trouvailles et d’analyses percutantes. Mais il reste que l’auteur confond complètement propagation de la gravité et propagation des ondes gravitationnelles, COMME A TOUS LES NIVEAUX DE LA VULGARISATION que nous avons explorés plus haut !
 
– à noter que le Centre National de ressources textuelles et lexicales, un dico quoi, dans sa définition de « gravifique »
 
 
affirme que c’est un synonyme vieilli pour gravitationnel
 
C’est tout à fait faut, cet adjectif était clairement employé par les anciens auteurs pour désigner les ondes ou autres vecteurs qui étaient capables de TRANSPORTER et d’appliquer l’effet gravitationnel d’un corps source à un corps cible. C’est d’ailleurs pourquoi il a été abandonné dans l’usage actuel, car la Théorie de la Relativité Générale pose, bien sûr, l’absence de ces ondes ou de ce vecteur, puisqu’elle attribue la gravitation à la déformation de l’espace.
 
Comme on l’a vu dans cet article, gravitationnel est beaucoup plus vague et est appliqué à tort, selon moi, aux ondes D’ORIGINE gravitationnelle, qu’on ferait mieux d’appeler ondes SPATIO-TEMPORELLES (même si ça fait un peu STARTREK…).
 

********************************************

Mise à jour d’août 2019

J’ai affirmé que c’était la théorie officielle et qu’il n’y a pas de différence de calcul de la trajectoire des planètes mais il est toujours bon d’en donner des exemples, en voici un :

http://theconversation.com/einsteins-theory-of-gravity-tested-by-a-star-speeding-past-a-supermassive-black-hole-100658

Dans cet autre site, le rédacteur reconnaît le fait mais conclut en disant que la seule différence, c’est qu’avant la gravité était instantanée mais que maintenant (vous l’avez deviné) elle se déplace à la vitesse c… (les planètes, elles, ne sont pas au courant)

http://astronomy.swin.edu.au/cosmos/N/Newton%27s+Second+Law+of+Motion

Ici, le Dr.Sten Odenwald répond à la question : « Est-ce que la gravité voyage plus vite que la lumière ? »

https://einstein.stanford.edu/content/relativity/q1510.html

Je ne résiste pas au plaisir de traduire sa réponse :

« Non, on peut dire que non. L’ équation d’Einstein de la Relativité Générale pour la gravité exige que la gravité voyage à la vitesse de la lumière, autrement les estimations que nous aurions pour  la courbure de la lumière et la précession du périhélie de Mercure seraient un peu différentes de ce que nous mesurons. Aussi, si nous envoyons un vaisseau spatial faire un grand voyage dans le système solaire, et là le délai de voyage de la lumière peut être de plusieurs heures. Si la gravité voyageait à une vitesse autre que la lumière, les trajectoires qui sont calculées pour vaisseau (oui il manque un mot) seraient toujours fausses parce qu’elles incluent des influences gravitationnelles qui changent tout le temps tandis que le vaisseau, le Soleil et les planètes bougent. On ne pourrait jamais arriver à faire les calculs avec tous les délais d’arrivée des influences gravitationnelles qui se combinent dans la trajectoire mesurée du vaisseau »

Oui c’est la réponse officielle à Harvard ou Stanford de ce gars : https://en.wikipedia.org/wiki/Sten_Odenwald.

Comment il a fait pour rentrer là oui il est alors qu’il n’arrive pas à accéder aux formules (ou aux simples constatations) qui font que le calcul correct de l’avance du périhélie de Mercure n’a rien à voir avec l’introduction d’une durée qu’il faudrait à la gravité pour atteindre Mercure ??? (voir Remarque déjà citée)

Mais nous sommes, rappelons-le, sur une page qui veut montrer que des lecteurs, des vulgarisateurs et même des chercheurs confondent et mélangent gravité et ondes gravitationnelles hein ? Alors le comble du bonheur, c’est l’envoi de notre ami très agité Julio//Julien qui nous a envoyé :

https://journals.aps.org/prl/pdf/10.1103/PhysRevLett.119.161102

Cet article de 2017 est la démonstration même de ce que j’avance. Ces trois chercheurs n’arrivent pas à se stabiliser mentalement entre des recherches sur les Ondes Gravitationnelles (ce qu’ils ont fait réellement à partir des données LIGO et Virgo) et des recherches sur la gravité. L’alternance commence entre le titre :

Bounding the Speed of Gravity with Gravitational Wave Observations

et le résumé :

The time delay between gravitational wave signals arriving at widely separated detectors can be used to
place upper and lower bounds on the speed of gravitational wave propagation. Using a Bayesian approach that
combines the first three gravitational wave detections reported by the LIGO Scientific and Virgo
Collaborations we constrain the gravitational waves propagation speed cgw to the 90% credible interval
0.55c<cgw < 1.42c, where c is the speed of light in vacuum. These bounds will improve as more detections
are made and as more detectors join the worldwide network. Of order 20 detections by the two LIGO detectors
will constrain the speed of gravity to within 20% of the speed of light, while just five detections by the LIGOVirgo-Kagra network will constrain the speed of gravity to within 1% of the speed of light.

Tout l’article est à l’avenant, un coup je te vois, un coup je te vois pas.

Je pourrais faire, en ce janvier 2020, une nouvelle mise à jour, mais je préfère faire un nouvelle article, une suite, la voilà :

kn0l.wordpress.com/la-gravite-est-elle-instantanee-ou-le-cafe-seulement-2/

48 commentaires

Comments RSS
  1. paul

    Très intéressant. J’ai l’impression aussi que cette recherche de graviton ne peut pas aboutir.
    On sait que l’espace temps est déformé et les corps qui s’y trouvent tombent dans une trajectoire qui est le fruit de cette déformation et cela sans latence.
    Donc pas de vitesse de la lumière pour la déformation de l’espace temps (ou gravité) mais bien plus, instantané?

  2. VALANCOGNE andre

    Je ne suis qu un beotien de la physique. Mais on evite ainsi la rapide programmation qui avec le temps perturbe l innovation (d ou l interet qu il faut porter a la jeunesse!).

    La question merite bien d etre posee.

    Il y a effectivement le plus souvent confusion entre « champ gravitationnel » et « ondes gravitationnelles ».
    C est le meme probleme entre un « champ electrique » ou sa variante relativiste le « champ magnetique » et « l onde electromagnetique ». Y a t il propagation du « champ electrique » ou le « champ » est il superposable a la « charge electrique »? Y a t il propagation du « champ gravitationnel » ou le « champ » est il superposable a la « particule massique »?
    La « particule » peut tout a fait etre consideree comme se superposant a son « champ ». Il n y a pas de « particule ponctuelle ».
    L integration de la masse doit se faire sur le champ entier comme la charge electrique.
    Il est vrai que le champ decroit rapidement avec la distance (1/R2) et donc sa densite.
    Ceci est parfaitement compatible avec la mecanique quantique ou la theorie de la gravitation et coincide avec ce qu on a denomme l intrication quantique.
    Je crois que deja un physicien iranien, ancien ministre de monsieur Moosadegh (qui avait nationalise le petrole de son pays et fut vite, sous la pression etrangere, demissionne par le shah) avait vu et ecrit cela…
    Cf une publication, il me semble d abord en anglais, lors d un sejour aux Etats Unis, puis en bon francais dans la revue de Monsieur Louis de Broglie lors d un sejour en France, dans les annees d apres guerre. Mais elle ne fut jamais citee.

    AVLYON

    • stefjourdan

      Merci de votre commentaire. Il y a effectivement des points communs entre les « champs » G et les champs M mais je ne m’avancerai pas plus sur le deuxième sujet vu que je n’y connais rien (à part qu’il est de notoriété que les champs magnétiques agissent instantanément…d’ailleurs la notion de champ a peut-être été inventée en partie pour « justifier » cette instantanéité : puisque le champ (produit par une entité « active ») s’étend déjà partout, au moment où la particule « passive » arrive dedans, la question de l’instantanéité ne se pose pas…. C’est peut-être aussi dans cette optique qu’Einstein a remplacé la Force d’attraction par un champ…
      Oui comme vous l’avez vu à la lecture de mes deux articles, la confusion se rencontre non seulement dans la basse vulgarisation mais dans la haute et jsuque dans les articles de chercheurs. Elle existait peut-être en partie même chez Einstein vu sa réponse (je ne sais plus si je l’ai incluse ici???)
      Vous écrivez « Y a t il propagation du « champ gravitationnel » ou le « champ » est il superposable a la « particule massique »?
      La « particule » peut tout a fait etre consideree comme se superposant a son « champ ». Il n y a pas de « particule ponctuelle ».
      L integration de la masse doit se faire sur le champ entier comme la charge electrique. »
      c’est exactement mon idée ! Si on visualise un atome avec un oeil chimique, on voit l’atome tel qu’on le connait, limité par « l’orbite » de ses électrons, mais si on le voit avec un oeil physique, il est beaucoup plus grand, infini même.
      Ce n’est pas la Terre qui attire la Lune, ce sont chaque atome de la Terre qui attire chaque atome de la Lune !

  3. Julio

    Bonjour stefjourdan

    Tout d’abord je veux préciser qu’il n’y a aucune agressivité dans mes propos même si je vous contredis.
    Vous semblez très fermé et ne croyez qu’en vos explications, alors je réponds une dernière fois à vos erreurs, libre à vous de continuer à vous croire le seul détenteur de la vérité contre toute la communauté scientifique, c’est dommage il suffirait de vous documenter par de bons bouquins de vulgarisation pour mettre fin au débat.

    1 – C’est faux, sans Einstein le calcul de la trajectoire de ne serait-ce que nos satellites serait faussé et le gps ne pourrait fonctionner par exemple.
    Les équations de Newton ne sont que des APPROXIMATIONS ! D’excellentes approximations à échelle humaine, mais l’erreur devient trop importante en astronomie.
    Vous n’ignorez pas que seule la relativité générale a permis de calculer la trajectoire réelle de mercure puisque vous l’évoquez, comment pouvez vous alors continuer de dire que Newton est toujours valable partout ? Et comment pouvez vous ignorer la réalité ou pire peut être mentir en affirmant « C’est avec les formules Newtonienne/keplerienne que sont calculés tous les phénomènes astromomiques futurs » ?? Bien sûr que non et vous ne pouvez l’ignorer (première conséquence très simple le gps).
    L’idée de force instantanée agissant à distance ne colle pas avec le relativité, vous ne l’ignorez pas non plus.
    C’est pour toutes ces raisons que les lois de Newton ne sont objectivement pas valides, ce ne sont encore une fois que d’excellentes approximations à petite échelle et loin du domaine relativiste, mais elles restent des approximations.

    Einstein explique bien la gravité par la courbure de l’espace temps ! Là où Newton disait que les corps s’attiraient sans explication, Einstein explique que s’ils semblent s’attirer c’est en fait leur trajectoire qui est modifiée dû à la courbure de l’espace temps, ils suivent des géodésiques déformées. Et Einstein ne parle pas que de corps, mais de tout ce qui contient de l’énergie au sens plus large. C’est bien une explication du phénomène observé.
    Je ne comprends pas comment vous pouvez dire le contraire si vous dites avoir bien compris cela …
    Et c’est bien grâce à la relativité générale qu’on peut expliquer que l’action de la gravitation n’est pas instantanée, ce qui posait évidemment problème avec l’idée de Newton encore une fois.
    La gravitation est une propriété de la géométrie de l’espace temps déformé par les corps qui induit des géodésiques non rectilignes.
    En d’autres termes, espace temps déformé par la densité de la matière (pour être parfaitement précis il faut parler d’énergie et non de masse comme le fait Einstein), qui en conséquence induit un déplacement des objets énergétiques. C’est pour cela que la gravitation est une accélération.
    Bien sûr si vous voulez contredire Einstein vous pouvez toujours …

    2 – Je maintiens, votre cqfd ne marche pas, le raisonnement est biaisé. La masse d’un trou noir est mesurable par la déformation de l’espace temps qu’il produit. Cela ne veut absolument pas dire qu’une information « sort » du trou noir !!! Aucune particule ne peut ressortir une fois passée l’horizon des évènements, mais il n’y a pas de contradiction avec la gravité ! C’est la déformation de l’espace temps qu’on peut observer, ce n’est pas une information venant du trou noir, c’est un peu comme une méta donnée et non une information intrinsèque au trou noir, c’est complètement différent. Un peu comme le contenu et le contenant.

    « Non désolé ça n’existe pas » ?? Ayez l’honnêteté intellectuelle de bien vouloir vous renseigner, même si je comprends bien que cela vous ennuie de voir votre théorie s’écrouler, mais la question n’est pas d’avoir tort ou raison, je vous félicite sincèrement pour votre article, vraiment, et on a le droit de se tromper tant qu’il n’y a pas de preuve de la validité d’une théorie, par contre c’est de la malhonnêteté que de refuser la réalité une fois les preuves établies.
    Voici l’étude des Chinois
    https://link.springer.com/article/10.1007/s11434-012-5603-3
    Je vous laisse le soin de chercher les confirmations qu’il y a eut par la suite.

    3 – Pardon mais je ne voulais absolument pas être agressif en disant simplement « ça semble vous étonner »

    Je me répète, j’admire et je félicite votre travail de recherche et d’explication de votre vision et compréhension de la gravitation et des ondes gravitationnelles !!
    Mais il faut savoir remettre en question ses idées parfois surtout quand on se retrouve seul, sans preuve, contre la science qui elle en a fournit à plusieurs reprises ces dernières années sur ce sujet.
    Toutes les théories sont bonnes, des milliers de physiciens élaborent des tas de théories très intéressantes, la votre aurait pu s’avérer juste, malheureusement déjà quelques années avant votre article des preuves ont montré que la gravitation se propage à la vitesse de la lumière, et que les ondes gravitationnelles sont des produits de la gravité.

    À vous de voir si vous voulez rester dans le siècle de Newton et l’ignorance des découvertes contemporaines, ou accepter les preuves établies et l’état actuel des connaissances scientifiques.

    Bien à vous

    • stefjourdan

      « Vous semblez très fermé et ne croyez qu’en vos explications, alors je réponds une dernière fois à vos erreurs, libre à vous de continuer à vous croire le seul détenteur de la vérité contre toute la communauté scientifique, c’est dommage il suffirait de vous documenter par de bons bouquins de vulgarisation pour mettre fin au débat. » Voilà un bon exemple de phrase très agressive. J’ai certainement lu, ces 50 dernières années, plus de vulgarisation que vous.

    • stefjourdan

      « C’est faux, sans Einstein le calcul de la trajectoire de ne serait-ce que nos satellites serait faussé et le gps ne pourrait fonctionner par exemple » Vous confondez/mélangez deux choses 1 la trajectoire des satellites, parfaitement calculables avec les lois de Kepler/Newton (mais qui doivent être remise à jour périodiquement car la Terre n’est pas une masse ponctuelle), ça n’a rien à voir avec la relativité. Ces calculs opèrent avec une précision insuffisante pour que des effets relativistes puissent les affecter. Les satellites ne sont pas en fait des objets relativistes (masses et vitesse trop faibles) par contre oui, 2 le comportement des horloges atomiques embarquées est affecté par leur vitesse et la diminution du champ gravitationnel DES LORS QU’ON les compare avec des horloges atomiques au sol. Cela tient à la précision insensée de ces machines, en fait elle détecteraient très bien la différence d’écoulement du temps entre le pied et le sommet de la tour Effeil, pas la peine de les mettre dans un sattelite !

    • stefjourdan

      « Vous n’ignorez pas que seule la relativité générale a permis de calculer la trajectoire réelle de mercure puisque vous l’évoquez, comment pouvez vous alors continuer de dire que Newton est toujours valable partout ? » Non, vous confondez « simulation » et « explication » : la trajectoire de Mercure est connue et prédite depuis bien avant la RG. Ce que la théorie de Newton n’expliquait pas, c’est la précession totale du périhélie. Elle expliquait seulement une précession « normale » due aux perturbations des autres planètes, de l’aplatissement du soleil, que sais-je. La RG permet de calculer la précession « finale » (somme de toutes les causes de précession) en y ajoutant une chose non prévue par la mécanique classique : Mercure, compte tenu de sa proximité au soleil et de sa vitesse, est un objet relativiste (contrairement aux autres planètes, pour lesquelles les effets relativistes doivent exister, mais resteront inférieurs à la précision des calculs donc invisibles/négligeables). Les calculs ‘keplero-newtoniens déjà permettaient non seulement de prévoir le mouvement futur des planètes mais de PREVOIR l’existence de planètes à partir des perturbations des autres : https://www.astronomes.com/lhistoire-de-lastronomie/la-mecanique-celeste. Calculs faits en prenant une gravité instantanée je vous le précise…

    • stefjourdan

      « Einstein explique bien la gravité par la courbure de l’espace temps !  » Ce n’est pas ce qu’on appelle une « explication » en Science, car la Relativité n’explique pas comment une masse « courbe » l’espace temps, et encore moins CE QU’EST « l’espace temps ». C’est donc une DESCRIPTION, pas une EXPLICATION.

    • stefjourdan

      « Et c’est bien grâce à la relativité générale qu’on peut expliquer que l’action de la gravitation n’est pas instantanée, ce qui posait évidemment problème avec l’idée de Newton encore une fois. » Oui et c’est bien grâce à la religion catholique qu’on peut expliquer que Jésus était le fils de Dieu.

      Au lieu de faire ce genre de phrase à la Science & Vie, trouvez-moi plutôt une source DE BASE de la RG parlant de la vitesse de la gravitation; Bonne chance, ça n’existe pas.

    • stefjourdan

      « – Je maintiens, votre cqfd ne marche pas, le raisonnement est biaisé. La masse d’un trou noir est mesurable par la déformation de l’espace temps qu’il produit. Cela ne veut absolument pas dire qu’une information « sort » du trou noir !!! Aucune particule ne peut ressortir une fois passée l’horizon des évènements, mais il n’y a pas de contradiction avec la gravité ! C’est la déformation de l’espace temps qu’on peut observer, ce n’est pas une information venant du trou noir, c’est un peu comme une méta donnée et non une information intrinsèque au trou noir, c’est complètement différent. Un peu comme le contenu et le contenant. » vous ne réfléchissez pas. Vous procédez uniquement par affirmations. « la masse du trou noir est mesurable par la déformation de l’espace temps qu’il produit ». ET alors, vous allez à côté du trou noir pour mesurer avec un mètre étalon la déformation de l’espace-temps ? non, vous observez le mouvement des corps à proximité de ce trou noir. Le champ de gravité ou « déformation de l’ET » est de toutes façons une information. Une mesure, c’est une information. A bon aucune « particule » ? mais à un autre moment, vous vouliez que la gravité vienne d’une particule, le graviton si je me souviens bien. Donc je pose ma question d’une autre manière : vos gravitons, comment ils sortent de l’horizon du trou noir ?

    • stefjourdan

      « Voici l’étude des Chinois https://link.springer.com/article/10.1007/s11434-012-5603-3 » Oui bien sûr, j’ai entendu parler de cet article. C’est évidemment du n’importe quoi. On ne peut pas mesurer la « vitesse de la gravité » en étudiant les marées. Déjà c’est embêtant, mais les marées sont dues à la lune et au soleil, c’est bien connu, donc en étudiant le retard des marées, on étudierait le temps que la gravité met à venir de la Lune ou du SOLEIL dites-moi ? Mais surtout ce genre d’étude est stupide. Il existe en effet une expérience PERMANENTE sur la « vitesse de la gravité ». C’est même une expérience idéale : la trajectoire des planètes autour du soleil, y compris la nôtre. Expérience idéale car dans le système solaire, on a des planètes très proches du soleil et d’autres EXTREMEMENT lointaines. Si la gravité se propageait à une vitesse finie, il serait facile de mettre en évidence le « retard » de la gravité sur les planètes les plus lointaines, qui ne « tourneraient » plus autour du soleil, mais autour du soleil là où il était des heures plus tôt, au moins 7 h (je n’ai pas la donnée exacte) pour Pluton. Donc plus on étudierait une planète lointaine, plus elle s’écarterait de la trajectoire Keplerienne, d’une ellipse parfaite. Mais ce n’est pas ce qu’on observe ! Toutes les planètes continuent très calmement et très sagement à suivre les orbites décrites par la mécanique pré-Einsteinnienne, c’est écrit dans tous les bouquins d’astronomie (sauf Mercure qui a une précession INFINITESSIMALEMENT inexplicable par Newton) .DONC si vous voulez absolument, parce que c’est votre croyance religieuse, affirmer que la gravité met du temps à atteindre les planètes lointaines, il faut alors que vous trouviez des forces correctrices (?) qui interviennent et s’arrangent pour l’accélération de leur trajectoire est orientée EN TEMPS REEL vers le soleil, sans aucun retard, d’où leur trajectoire parfaitement elliptique, c’est ce qu’essaie de faire l’auteur de Mathpages, site cité dans l’article, sans donner de références ou de calculs. (cette expérience « des planètes » a été commentée par Laplace, ET EINSTEIN était au courant, et donc il s’est bien gardé de parler de la vitesse de la gravité dans la RG, voilà, y’a pas un mot d’Einstein sur la « vitesse de la gravité », il n’était pas encore atteint pas la religion des gens qui « ont lu de la vulgarisation ». Cherchez plutôt ici ce qu’il a dit : https://einsteinpapers.press.princeton.edu/ (Je n’ai évidemment rien contre les chinois, MAIS 1 sur la forme : la théorie de la gravitation, les recherches sur la gravitation ne sont pas chinoises. Si il existait une expérience à partir des « marées » mesurées avec des accéléromètres ou même des balances et suceptibles de prouver la vitesse de la gravitation, ces expériences aurait déjà été inventées et FAITES en Europe au XVIIIè siècle je pense. Sa « découverte » n’a été reprise que sur certains sites et blogs mineurs, voire des journaux grand-public 2 sur le fond : le chercheur principal a une théorie spéciale sur la gravitation qu’il applique – à la vitesse de la gravitation – aux éclipses – aux sondes Pioneer etc. Par exemple au niveau des éclipses, il aurait découvert un effet non prévu par Einstein, si vous regardez. Donc Oh LA LA, ATTENTION. Je ne dis pas que sa théorie est fausse hein, j’ai pas tout lu, mais elle ne paraitpas un bon truc à citer pour défendre la Relativité. (moi par exemple je n’ai pas d’observations non-vues par Einstein ici. Je me contente de dénoncer des choses qu’à mon avis TOUT LE MONDE CROIT mais qu’Einstein n’a PAS ECRITES. Au niveau des sondes Pioneer, on a trouvé la raison du ralentissement, et c’est pas la sienne (matière négative de l’Univers ?), en fait c’est la pression de radiation de la matière fissile embarquée sur l’antenne. Le plutonium embarqué emet du alpha, du beta, du gamma et un ou deux des trois « appuie » sur l’antenne, or l’antenne est dirigée vers la Terre, donc grosso merdo, vers le soleil…)

    • stefjourdan

      « malheureusement déjà quelques années avant votre article des preuves ont montré que la gravitation se propage à la vitesse de la lumière, et que les ondes gravitationnelles sont des produits de la gravité. » NON, aucune « preuve » ne l’a montré. ET pourquoi parlez-vous soudainement des ondes gravitationnelles alors que nous discutons de la gravité ?
      La vitesse des OG ne fait aucun débat.

      • jjouffr

        Arrêtez de vous ridiculiser !
        Je vous pensais de bonne fois mais avec ces réponses que vous venez de faire je constate que j’ai perdu mon temps à essayer de vous éveiller à l’état actuel des connaissances.

        Votre mauvaise foi est assez exemplaire !!! Vous niez toutes les preuves qu’on peut bien vous apporter et faites votre salade avec de mauvais raisonnements uniquement pour étayer votre fausse vision de la réalité qui a pourtant été prouvé à maintes reprises et fait l’unanimité de la classe scientifique, mais votre château de cartes ne tient pas bien évidemment.

        Vous êtes fort !!! Très fort !! Plus fort qu’Einstein et de l’ensemble de la communauté scientifique ! Je ne comprends même pas comment on peut être d’une telle prétention !
        La vérité est que vous ne comprenez pas ce qu’un enfant peut comprendre, et vous vous obstinez dans une voie pour ne pas perdre la face.

        Je suis triste pour vous M stefjourdan
        Bonne continuation

    • stefjourdan

      Mon pauvre Julio. Pourquoi avez-vous changé d’alias ? La science, c’est l’esprit critique. Quand vous ête un vrai scientifique et que vous venez en contact avec des idées non-orthodoxes, alors vous devez les étudier. Ca fait partie du processus scientifique. Mais votre style et le contenu de vos interventions montre que le contact avec des idées critiques vous met dans un état de dissonnance cognitive. Il ne me viendrais pas à l’idée de dire à quelqu’un qu’il est ridicule dans un débat scientifique, ou que sa position me rend « triste ». Apprenez comment ça marche et…oui, allez perdre votre temps ailleurs, merci.

      • Julio

        Ne parlez pas de science ni d’esprit critique vous ne savez pas ce que c’est.
        Vous vous obstinez à déformer la réalité pour la faire coller à votre pseudo théorie dont on sait depuis longtemps qu’il en est tout autre.
        Je travaille au cnrs a Lyon, et j’ai comme voisins à l’étage du dessus des confrères de l’insu dont j’avais envisagé l’idée de vous mettre en contact avec l’un d’eux qui saurait plus précisément que moi mettre à défaut vos raisonnements biaisés et volontairement orientés mais après vos dernières réponses j’ai compris que c’était peine perdue, et que des gens comme vous niants toute évidente vérité unanimement établie et sans une once d’humilité sont incapables de se remettre en question et d’apprendre quoique ce soit.

    • stefjourdan

      Calmez vous, CALMEZ VOUS ! Je croyais que vous alliez ailleurs, pourquoi revenez vous ici vous agiter ? Je vais jeter un coup d’oeil à votre lien, merci.
      Mon avis : c’est un article sur la propagation des ondes gravitationnelles, pas de la gravité, deux choses différentes. Pas besoin de LIGO pour observer la seconde. Le titre qui mêle les deux concepts montre que l’auteur est dans la confusion mentale ou seulement rédactionnelle, mais c’est embêtant pour un scientifique.

      Titre : « Bounding the Speed of Gravity with Gravitational Wave Observations » = Mettre des bornes à la vitesse de la gravité grâce à l’observation d’ondes gravitationnelles »

      Résumé : « Using a Bayesian approach that combines the first three gravitational wave detections reported by the LIGO Scientific and Virgo Collaborations we constrain the gravitational waves propagation speed » = En utlisant une approche Bayesienne sur les trois premières détections d’ondes G par le LIGO et Virgo, nous fixons des bornes à la vitesse de propagation des ondes gravitationnelles »

      Donc vous voyez dans le titre, c’est la vitesse de la « gravité », dans le résumé, ce n’est plus la vitesse de la « gravité » mais la vitesse des « OG ». Ce n’est pas la même chose. Comment expliquez-vous cette dérive sémantique ? En science, les choses ont un nom et un seul : le voltage c’est du voltage, pas de l’intensité, par exemple. Si je rencontrais un papier sur l’électricité disant qu’il va étudier le voltage et qu’en cours de route il s’avère qu’il étudie l’intensité, ça serait embêtant. Ici c’est pire, on n’est pas dans le même sujet du tout !

      Pendant tout le reste de l’article, ils opèrent la même alternance entre « speed of gravitationnal waves » (ce qu’il ont réellement étudié) et « speed of gravity » (qu’on ne peut pas étudier avec le LIGO ou VIRGO, ça n’a pas de sens, ce sont des détecteurs d’ondes gravitationnelles, pas des détecteurs de gravitation. Des détecteurs de gravitation : votre balance, un accéléromètre, la trajectoire d’un corps pesant…)

  4. La main qui Ecrit

    Julio, t’es indécrottable. 🙂 Tu ne lis que ce que tu veux lire. J’ai parlé de 2 points de vue. J’honore aussi bien la découverte d’Einstein que celle de Newton. Extrait:

    La Gravité est un champ vectoriel.
    C’est-à-dire que le champ définit une direction : un centre.
    De même chaque point de l’espace varie selon sa distance.
    Ainsi, plus on se rapproche du centre, plus le champ est intense.
    Ainsi, plus on s’écarte du centre, plus le champ est faible.
    Toutefois, il ne disparaît pas pour autant.
    Il diminue en intensité, c’est tout.
    Ainsi la Gravité est un phénomène universel !

    Cette attraction se fait via une déformation de l’espace.
    Cette déformation modifie la trajectoire rectiligne des objets.
    Ainsi l’objet fonce tout droit… Dans une courbe !
    Ce qui signifie que sa trajectoire devient courbe.
    Ça c’est pour le point de vue d’Albert Einstein.

    De même, elle définit une force d’attraction.
    Une force qui croît ou décroît selon la distance.
    Une distance par rapport à un centre gravitationnel.
    Ça c’est pour le point de vue d’Isaac Newton.

    L’ensemble donne un espace courbe qui attire en son centre.
    Ces points de vue ne s’opposent pas, ils se complètent !

    • Julio

      Tu as tout juste !!! Jusqu’à 5 lignes de la fin … La relativité générale remplace Newton et ne le complète pas.
      La gravité ne définit pas de force d’attraction, en suivant cette idée on a une bonne approximation mais si on veut être très précis c’est faux.
      La gravitation n’est pas une force, c’est une déformation de l’espace temps comme tu le dis qui induit que tout ce qui a de l’énergie suit des géodésiques modifiées.
      Plus besoin de force !!!! Et donc plus besoin d’interaction entre les objets pour décrire la gravité.

      Aucune force n’attire les particules de masse nulle comme les photons pourtant ils sont soumis à la gravité !

  5. La main qui Ecrit

    Réponse à Julio : 1 ) « Vous avez tort de parler de « maintien de la validité de la théorie de Newton » » ??? Le fait qu’Albert définisse les lois de la relativité ne veut pas dire que Newton a tort. Ce sont des points de vue qui se complètent et permettent d’aborder le sujet selon des points de vue différent. Autrement dit il n’y a pas de vérité toute faite. Elle se laisse découvrir par les curieux. Les curieux : ceux qui cherche l’info sans tout gober tout de suite mais en analysant les points de vue comme ceux d’Albert ou d’Isaac. 2 La Gravité est un CHAMP VECTORIEL qui entraîne une déformation de l’espace. Cause et conséquence sont 2 choses différentes. Pour le point 3, je suis d’accord. Et pour ce qui est de la Gravité, elle est un phénomène instantané : toutes les étoiles au sein des galaxies se partagent la même information sur le centre gravitationnel galactique. Même les étoiles à l’extrémité. Et le seul moyen est que l’information soit instantanée. C’est pourquoi une galaxie est homogène ! Il suffit simplement de regarder les étoiles pour y trouver la réponse !

    • Julio

      Je te laisse lire mon ultime réponse à stefjourdan ci dessus.

      En tant que curieux qui ne gobe pas tout tout de suite tu sauras prendre connaissance des études (dont j’ai mis le lien de la première parue il y a 7 ans déjà …) démontrants que l’action de la gravité n’est pas instantanée, comme l’avait prédit Einstein.

    • Julio

      Arrêtez de vous ridiculiser !

      Je vous pensais de bonne fois mais avec ces réponses que vous venez de faire je constate que j’ai perdu mon temps à essayer de vous éveiller à l’état actuel des connaissances.

      Votre mauvaise foi est assez exemplaire !!! Vous niez les preuves qu’on peut bien vous apporter et faites votre salade avec de mauvais raisonnements uniquement pour étayer votre fausse vision de la réalité qui a pourtant été prouvée à maintes reprises et fait l’unanimité de la classe scientifique, mais votre château de cartes ne tient pas bien évidemment.

      Vous êtes fort !!! Très fort !! Plus fort qu’Einstein et l’ensemble de la communauté scientifique ! Je ne comprends même pas comment on peut être d’une telle prétention !
      La vérité est que vous ne comprenez pas ce qu’un enfant peut comprendre, et vous vous obstinez dans une voie pour ne pas perdre la face.

      Je suis triste pour vous M stefjourdan
      Bonne continuation

  6. Jm Willem

    Bonjour, félicitations pour ces longues analyses. De plus, le style pour le moins chatouilleur contraste avec une certaine austérité monotone de beaucoup d’articles. Vous avez le mérite de faire réfléchir, de faire prendre du recul sur ce qui s’écrit en général. Et comme vous le soulignez, la méga-connaissance de certains dans les mathématiques hyper-pointues, ne doit pas escamoter parfois de grosses failles du bon sens. Je serais heureux de pouvoir lire d’autres publications de votre part.

    • stefjourdan

      Bonjour et merci de cette appréciation. En physique, je n’ai pas beaucoup d’artiicles, mais j’en ai plein d’autres comme vous voyez, en des matières différentes. La logique peut s’appliquer partout, à condition tout de même de débroussaiiller le sujet avant de s’y aventurer. La situation actuelle de la physique est très étonnante, avec cette confusion OG/gravitation qui semble régner dans tous les cercles !

  7. Julio

    Bonjour
    article très intéressant et beau boulot !! Par vous argumentez sans preuve sur la théorie de la gravité à effet immédiat !?
    Ça m’intéresserait très fortement d’avoir des références sur le sujet parce que je ne trouve rien et au contraire que des articles (comme ceux que vous critiquez) qui parlent de propagation de l’action de gravité à la vitesse de la lumière.

    Par ailleurs je me permets d’indiquer que ce n’est pas parce que les trous noirs ont une action de gravité que ça prouve que la gravité se propage à une vitesse supérieure à celle de la lumière ou instantanément, c’est un faux argument, ça dépendrait complètement de la nature de l’hypothétique graviton.

    Julien

    • stefjourdan

      Bonjour et merci de votre intérêt.
      1 je n’argumente pas sans preuves, loin de là. La principale démonstration, à la vue de tous, est le maintien de la validité de la théorie de Newton pour les calculs effectifs. Or cette théorie considère évidemment la gravité comme instantanée. Le principal développement de mon article est la confusion mentale à tous les niveaux de la vulgarisation entre gravité et ondes gravitationnelles, donc aussi une preuve par l’absurde.
      2 Vous n’avez pas compris (peut-être qu’elle est mal rédigée) la question des trous noirs : si la gravité avait une vitesse égale à celle de la lumière, piégée dans le trou noir par définition, alors la gravité serait piégée aussi, par hypothèse, ce qui n’est évidemment pas le cas. La gravité sort sans difficulté du trou noir, comment fait-elle ?
      3 le graviton est et restera hypothétique donc il n’aura jamais de « nature ». Les bosons de jauge agissent dans le domaine de l’infiniment petit. Penser qu’ils pourraient agir à l’infini est une extention naîve de la théorie quantique.Par ailleurs il faudrait qu’ils interagissent avec la matière tout en continuant leur chemin ! (la lune est toujours attirée par le soleil pendant une occultation : les gravitons issus du soleil traverseraient la terre pour aller intéragir avec les atomes de la lune ????)
      4 le meilleur article pour aller plus loin est celui (et les autres du même site) dont j’ai extrait l’image et donné l’adresse, sur mathpages….lire aussi mes réponses aux autres commentaires….

      • Julio

        Bonjour

        Je pensais avoir répondu à votre message il y a déjà un bon moment …. Désolé de cette réponse tardive du coup.

        En réponse à vos points :
        1 – Vous avez tort de parler de « maintien de la validité de la théorie de Newton », on l’utilise par souci de simplicité lorsque cette approximation nous suffit, mais ça reste une formule approximative qui fonctionne comme vous l’avez indiqué pour des masses et donc attractions relativement faibles. Newton n’explique pas ce qu’est la gravité, Einstein oui !! Et vous ne semblez pas avoir compris cette explication de déformation de l’espace temps lui même !! (J’y reviens dans les points suivants)
        Vous ne pouvez pas garder Newton pour preuve en parlant de relativité puisque c’est justement la relativité qui a montré que Newton avait tort (au mieux sa formule tombe juste si on n’a pas besoin d’être précis)

        2 – Oui j’avais bien compris, mais la gravité EST une déformation de l’espace-temps, ce n’est pas « quelque chose » qui évolue dans cet espace-temps (vous ne pouvez donc pas dire « si la gravité sort du trou noir » ça n’a pas de sens), donc ça n’a absolument rien à voir avec un photon qui est une particule qui se déplace à une vitesse donnée dans l’espace-temps. Donc aucune particule ni objet qu’on connaisse de l’univers ne peux échapper à l’attraction d’un trou noir lorsque cet objet est suffisamment proche, même la lumière (photon), mais la masse d’un trou noir déforme l’espace-temps et cette déformation évolue jusqu’aux confins de l’univers à la vitesse de la lumière (si si ça y est ça a été mesuré, je ne le savais pas lorsque j’ai lu votre article mais on le mesure en comparant l’effet de la gravité du soleil et de la lune sur la déformation de la terre et donc les marées) et se fait ressentir partout dans toutes les directions.
        L’espace-temps lui-même peut d’ailleurs se déformer et engendrer des éloignements des objets qu’il contient à des vitesses supérieures à celle de la lumière (c’est ce qui s’est passé lors de l’inflation après le Big Bang d’après la théorie actuelle) mais ces objets qui sont de fait éloignés à des vitesses supérieures à celle de la lumière ne se déplacent pas dans l’espace-temps c’est l’espace-temps qui s’agrandit en tous points.

        3 – Évidemment la lune est toujours attirée par le soleil pendant une occultation par la terre, pour la même raison il n’y a rien qui traverse la terre c’est l’espace-temps lui-même qui est courbé à la fois par la masse du soleil et la masse de la terre tout comme si vous placez une grosse balle sur un trampoline toutes les balles que vous placerez sur ce même trampoline seront attirées vers la grosse mais rien ne traverse le trampoline entre ces balles, qui d’ailleurs comme l’espace-temps pour la gravité se déforme à une certaine vitesse qui se trouve être la vitesse de la lumière la même vitesse que les ondes gravitationnelles qui sont les conséquences de perturbations de cet espace-temps.
        Ça semble vous étonner que des particules puisse traverser un astre ? C’est pourtant le cas des neutrinos, vous êtes traversé chaque seconde par plusieurs milliards de neutrinos par centimètre carré de votre peau ! Donc si une hypothétique particule qui venait à être découverte serait l’image de la déformation de l’espace-temps, il se peut très bien que cette particule à l’image du neutrino n’interagisse que très rarement avec la matière et donc traverse tout.

      • stefjourdan

        Je ne trouve votre longue réponse qu’aujourd’hui 1 Non je n’ai pas « tort ». Ce maintien de la validité des calculs de Newton est exprimé partout. C’est avec les formules Newtonienne/keplerienne que sont calculés tous les phénomènes astromomiques futurs. Newton n’explique pas la Gravité, Einstein non plus. Les deux « décrivent » son action sans avoir de mécanisme. Ne discutez pas avec quelqu’un qui a beaucoup travaillé sur la question en commençant pas l’accuser de ne pas avoir « compris ». Je comprends très bien l’idée de déformation de l’espace temps. Attaquez-vous plutôt à ceux qui, comme je le montre, n’ont effectivement rien compris, et mélangent à qui mieux mieux ondes gravitationnelles et gravité.
        2 J’écris que la gravité « sort » du point noir pour montrer que les affirmations « rien ne sort du point noir » sont fausses. Par rien ils entendent évidemment « aucune information », or le champ de gravité d’un point noir est une information sur la masse de ce point noir et cette information « sort » de l’horizon « dont rien n’est censé sortir » CQFD. Le reste de votre point sur la mesure de la vitesse de la gravité par observation des marées, non désolé, ça n’existe pas. 3 Ca ne « semple pas m’étonner » (soyez moins agressif) J’utilise que le fait que la gravité « traverse » les masses comme fait contradictoire avec les théories qui veulent affecter une particule interragissante à la gravitation. Bien sûr que les neutrinos traversent la Terre (ou n’importe quelle autre planète/étoile) mais justement, ils n’interragissent pas, d’où la difficulté de leur détection. Tout ce qui interragit est stoppé, c’est le principe même. Effectivement, si la gravitation est une déformation de l’ET, alors elle n’interragit pas avec la matière, au sens corpusculaire, d’où la transparence des astres, un bon point pour la Relativité Générale car il est très difficile d’imaginer des gravitons, en nombre incroyablement nombreux (imaginez par ex. le nombre d’atomes de votre corps et le nombre d’atome de la Terre et déduisez-en le nombre de graviton qui devraient se croiser à chaque instant rien que pour produire « votre poids », c’est idiot)

    • La main qui Ecrit

      « Boson » de Higgs = ? Particule de Higgs = spin 0. boson = spin 1. Si les scientifique ne sont pas foutu de faire la différence entre un spin 0 et un boson, laissez tomber pour le graviton de spin 0. Spin 0 = je suis instantané, omnidirectionnel, universel et omniprésent dans l’espace ! Je sors sous peu un livre qui explique le tout, y compris le spin 0 ainsi que le graviton. Il ne se déplace pas, il est immobile comme la particule de Higgs appelée faussement un « boson ». C’est l’information qui est véhiculée instantanément. Nos hommes de sciences sont à la traîne. En fait, ils sont perdus mais ils n’ose pas le dire. Onde spatio-temporelle = mouvement dans une Gravité. Elle n’a rien à voir avec la perte de masse des trous noirs. Mais comme ils ne savent pas l’expliquer, ils mettent tout sur les dos de « l’onde gravitationnelle ». Pourtant, ils savent très bien que ce n’est pas la perte de masse qui crée l’onde mais le MOUVEMENT ! Ils se contredisent eux-même. La perte de masse à rapport avec un barogénocide de la matière noire. Rien à voir avec l’onde. Ils sont perdus. Je vais les aider à y voir plus clair dès que mon livre est enfin prêt. Encore un mois.

      • Julio

        Et hop !!! Einstein et la relativité générale à la poubelle !!🤣
        Bonne chance pour votre livre !😉

      • stefjourdan

        Un peu fouilli, mais merci de votre intervention et bonne chance pour votre bouquin.

    • stefjourdan

      Finalement, vous m’avez envoyé un article, à mon tour de vous en faire passer un, puisque vous lisez l’anglais. Ce n’est pas un article de recherche, mais de vulgarisation d’assez bon niveau : http://theconversation.com/einsteins-theory-of-gravity-tested-by-a-star-speeding-past-a-supermassive-black-hole-100658 Il est intéressant par rapport à ce dont nous avons discuté dans ces phrases : « For most places in the universe, where gravitational fields are weak, the mathematics of Newton and Einstein give identical results for the motion of galaxies, stars and planets. » « Their orbits should therefore agree with both Newton and Einstein. » . Donc comme je vous le disais, c’est l’enseignement officiel. La trajectoire des planètes ou des étoiles,(j’ajouterai des satellites) n’est pas modifiée par rapport aux calculs de Newton, tant que la situation n’est pas « relativiste » (très grandes masses à proximité, très grande vitesse). Bon le vulgarisateur est Geraint Lewis, Professor of Astrophysics, University of Sydney. Pas ma concierge. Bien à vous

      • Julio

        Bonsoir stefjourdan

        Je ne comprends pas en quoi de tels propos vous rassurent ? Je vous réponds, mais les exemples de publication que je vous ai transmis concernent des preuvres de la mesure de la vitesse de propagation de la gravité, ce qui est le sujet de votre article. Je serais très intéressé de voir des publications non contestées indiquant que la gravité se propage instantanément.

        OUI les lois de Newton sont valables et largement utilisées je n’ai jamais dit le contraire, mais il faut distinguer les usages et la théorie mathématique.
        À l’usage, pour calculer l’orbite de la terre autour du soleil Newton fonctionne très bien, à condition qu’on ne recherche pas une précision à 20 décimales après la virgule !

        MAIS NON d’un point de vu strictement mathématique il n’y a pas égalité entre les formules de Newton et celles d’Einstein, quelque soit l’intensité du champ gravitationnel.
        Oui dans des champs gravitationnels faibles l’erreur avec les équations de Newton est infime et donc très souvent négligeable pour la plupart des usages.

        C’est exactement comme dire qu’une galaxie lointaine n’a aucune influence gravitationnelle sur la terre, c’est mathématiquement et factuellement faux mais dans la pratique l’effet est tellement dérisoire et impossible à mesurer tant il est faible que personne ne vous en voudra de le dire.
        Pourtant selon la théorie actuelle l’effet de la gravité n’est par limité dans l’espace, il s’atténue proportionnellement avec le carré de la distance séparant deux objets mais n’est jamais nul.

        En disant que les équations de Newton sont fausses car approximatives, j’adopte un point de vu purement mathématique et théorique, mais si on me demande de calculer l’attraction terre/lune je les utiliserai bien volontier comme tout le monde !!

      • stefjourdan

        « Je ne comprends pas en quoi de tels propos vous rassurent ? » Vous vous intéressez à l’astrophysique ou à la psychologie ? je n’ai pas besoin d’être « rassuré », vous peut-être ?

        « …publication que je vous ai transmis concernent des preuvres de la mesure de la vitesse de propagation de la gravité » NON, comme je vous l’ai montré, cette publication concerne seulement la propagation des ONDES GRAVITATIONNELLES. Mais les auteurs sont affligés d’un problème commun, bien développé dans mon article, qui fait qu’ils emploient tantôt « gravitational waves » et tantôt « gravity » or ce sont deux choses différentes. Vérifiez par vous même l’existence de ces deux choses séparées (par Ex. Einstein a-t-il prédit l’existence d’ondes G. ou bien a-t-il prédit l’existence de la gravité ?) et relisez ensuite l’article en question mot à mot.

      • Julio

        « Vous vous intéressez à l’astrophysique ou à la psychologie ? je n’ai pas besoin d’être « rassuré », vous peut-être ? »
        Je vois que vous n’avez toujours pas mis de côté voir agressivité … 😏
        Si je vous demande pourquoi vous parlez encore d’un autre sujet que le fond de votre article en relevant cette phrase dans votre message c’est que vous ne cessez de changer de sujet (mille feuille argumentatif) et vous accrochez à de petites phrases à droite à gauche en faisant des liens qui n’ont pas lieu d’être.
        Si je voulais faire la même chose je vous montrerais cette conclusion de l’article « Until that day, Einstein’s vision of space and time reigns supreme. »
        Vous n’avez d’ailleurs pas répondu à mon explication en préférant l’agressivité, si cela pouvait indiquer que vous étes d’accord j’en serais fort ravi pour vous !

        « les auteurs sont affligés d’un problème, sujet aussi de mon article, qui fait qu’ils emploient tantôt « gravitational waves » et tantôt « gravity » or ce sont deux choses différentes »
        Peut être que si l’ensemble des publications sur le sujet ne vous conviennent pas c’est votre interprétation qu’il faudrait remettre en question et non la communauté scientifique ?

        J’ai bien lu l’article, et il ne fait aucunement mention d’ondes gravitationnelles, ni dans les termes, mais admettons que l’auteur soit trop « brouillon » (pour ne pas dire stupide…) pour faire la confusion avec la gravité rien dans l’article n’y fait penser, le sujet est bien les équations d’Einstein sur la gravité validées aux abords de ce trou noir.

        « Einstein a-t-il prédit l’existence d’ondes G. ou bien a-t-il prédit l’existence de la gravité ? » (Je ne vais pas jouer sur les mots mais il n’a pas prédit l’existence de la gravité, il en explique la cause pour mieux coller aux observations que Newton)
        Ne faites pas de procès à Einstein pour ce qu’il n’a pas dit ! Il a travaillé sur la relativité générale pour pallier l’incohérence entre les lois de Newton et la relativité restreinte. Ce n’est pas dans une théorie qu’on évoque toutes les conséquences physiques de celle ci.
        Par contre, cela ne veut pas dire qu’il n’y a pas de conséquences observationnelles de sa théorie, quand bien même il n’aurait même pas imaginé certaines.
        C’est ce qui se passe depuis des décennies, un jour sans doute une observation montrera une lacune de ces théories mais pour l’instant l’observation de divers phénomènes en apportent des validations concrètes.
        Les ondes gravitationnelles sont l’infime trace visible de la courbure de l’espace temps qu’à prédite Einstein. Peu importe que dans ses publications ce phénomène soit évoqué ou non.

        D’ailleurs on n’a pas fini de découvrir des applications encore inconnues à ce jour de ses théories, ce n’est pas moi qui le dit je vous rassure mais je l’ai entendu de plusieurs physiciens, illustrants à quel point ses théories sont profondes et nous dépassent encore.

        « Vérifiez par vous même l’existence de ces deux choses séparées »
        Tout ce que j’ai pu lire sur le sujet va dans le sens du consensus que vous semblez nier, à savoir en deux mots :
        – masse déforme l’espace temps -> mouvement des corps énergétiques affecté par cette déformation (gravité)
        – on appelle la variation de la déformation de l’espace temps des ondes gravitationnelles
        Toutes les lectures scientifiques sur ces sujets sont cohérentes.

      • stefjourdan

        « Je vois que vous n’avez toujours pas mis de côté voir agressivité … 😏
        Si je vous demande pourquoi vous parlez encore d’un autre sujet que le fond de votre article en relevant cette phrase dans votre message c’est que vous ne cessez de changer de sujet (mille feuille argumentatif) et vous accrochez à de petites phrases à droite à gauche en faisant des liens qui n’ont pas lieu d’être. »
        CHANGER DE SUJET ? Je parle de la même chose depuis le début. Vous êtes un expert dans les phrases creuses. Vous pourriez devenir homme politique.

        Si je voulais faire la même chose je vous montrerais cette conclusion de l’article « Until that day, Einstein’s vision of space and time reigns supreme. »

        Je veux bien voir ça.

        « Vous n’avez d’ailleurs pas répondu à mon explication en préférant l’agressivité, si cela pouvait indiquer que vous étes d’accord j’en serais fort ravi pour vous ! »

        Répondu à votre « explication » ? Je ne sais pas de quelle « explication » vous voulez parler. Je n’ai pas un plein temps disponible pour me pencher sur vos accusations et c’est plutôt vous qui ne répondez pas à des questions simples. Faites des envois plus courts SVP si vous voulez que je réponde à un point précis, pas comme vos romans fleuves.

        Peut être que si l’ensemble des publications sur le sujet ne vous conviennent pas c’est votre interprétation qu’il faudrait remettre en question et non la communauté scientifique ?
        Chaque article/publication est à examiner de façon indépendante. J’en cite de nombreux dans mon article qui tiennent au moins la route.

        « J’ai bien lu l’article, et il ne fait aucunement mention d’ondes gravitationnelles, ni dans les termes, mais admettons que l’auteur soit trop « brouillon » (pour ne pas dire stupide…) pour faire la confusion avec la gravité rien dans l’article n’y fait penser, le sujet est bien les équations d’Einstein sur la gravité validées aux abords de ce trou noir. »

        HEIN, vous ne l’avez pas relu ! Je vais vous citer les phrases une par une, quand j’aurai le temps. Merci d’attendre :

        le titre déjà « Speed of Gravity with Gravitational Wave… » (à quel moment, qui a dit, ou pire prouvé que la vitesse de la gravité avait à voir avec la vitesse des ondes gravitationnelles ?)

        donc sans recopier tout l’article, ils ne cessent de passer de l’un à l’autre comme si c’était la même chose, gros gros problème car en physique, un nom c’est un nom. Imaginez un article actuel qui parlerait tantôt du champ électrique et tantôt du champ magnétique ? (cela doit exister, avant la découverte de la nature leur intrication

        je vous rappelle que « gravity » c’est ça : https://en.wikipedia.org/wiki/Gravity
        tandis que « gravitational waves », c’est ça : https://en.wikipedia.org/wiki/Gravitational_wave

        lisez les bien (il est possible toutefois qu’il y ait des passages où les rédacteurs ont fait la même confusion, mais au moins jusqu’à présent, il y a deux pages différentes, pour deux phénomènes différents)

        (Je ne vais pas jouer sur les mots mais il n’a pas prédit l’existence de la gravité, il en explique la cause pour mieux coller aux observations que Newton) hein, qu’est-ce que vous racontez ? vous n’êtes pas au courant que c’est Einstein qui a prédit l’existence des OG ? je vous montre la différence entre la gravité et les OG par cette boutade que E. n’a pas prédit la gravité.

        « Ne faites pas de procès à Einstein pour ce qu’il n’a pas dit ! Il a travaillé sur la relativité générale pour pallier l’incohérence entre les lois de Newton et la relativité restreinte. »
        Mais c’est quoi ces phrases de prof de français, je ne fais pas de « procès » à Einstein, je vous parle de ce qu’il a prédit. Vous êtes sourd ou mal-comprenant ? Lisez bien et essayez de comprendre cette phrase de l’article de WP : « They were proposed by Henri Poincaré in 1905[1] and subsequently predicted in 1916[2][3] by Albert Einstein on the basis of his general theory of relativity.[4][5]  »

        cette phrase est dans la page sur les OG, elle parle des OG, Einstein parlait des OG, c’est bon ? Un procès à Einstein pour ce qu’il ln’a pas dit, mais vous avez fumé quoi ?

        « J’ai bien lu l’article, et il ne fait aucunement mention d’ondes gravitationnelles, ni dans les termes, mais admettons que l’auteur soit trop « brouillon » (pour ne pas dire stupide…) pour faire la confusion avec la gravité rien dans l’article n’y fait penser, le sujet est bien les équations d’Einstein sur la gravité validées aux abords de ce trou noir. » Vous n’avez pas lu le bon article. Je parlais de celui que vous m’avez envoyé, quel autres voulez-vous ? https://journals.aps.org/prl/abstract/10.1103/PhysRevLett.119.161102

      • Julio

        Cher stefjourdan

        J’espère ne pas vous offusquer en ne rentrant pas dans votre jeu de basses attaques personnelles.
        Je préfère vous transmettre un dernier lien que vous aurez plaisir à écouter spécialement vers 18min14s.

        Si Gabriela Gonzalez elle même, éminente spécialiste de la question, ne peut vous faire entendre raison personne ne le pourra.

        Vous avez toutes les réponses à vos contradictions dans cette vidéo

        CQFD

      • stefjourdan

        « Basses attaques personnelles » ? Vous vivez dans un SitCom ? Je vais écouter ce que dit cette dame, en entier. « Eminente spécialiste » je ne sais pas, je vois que c’est la porte parole du Ligo https://en.wikipedia.org/wiki/Gabriela_Gonz%C3%A1lez Disons une prof. d’université qui a su se faire mousser dans ce travail d’enseignement et de communication. A première vue, les papiers cités dans son CV (accessible sur la même page) ne mélangent pas Ondes Gravitationnelles et Gravité, mais je verrai ce qu’elle nous raconte dans cette vidéo. Merci

        Voici ma revue de cette vidéo :

        Considérations générales :
        – c’est pénible d’écouter cette conférence en doublage, je n’ai pu continuer au delà de 22′. A la limite, si elle existe en VO je veux bien réessayer.
        – il s’agit d’une conférence grand-public, par une « attachée de relations publiques » prof de fac, pas une « éminente spécialiste de la question »
        – pour monter son niveau il est conseillé de regarder plutôt des vidéos de chercheurs, donc de première main ou encore mieux de lire leurs articles, comme ici : https://ned.ipac.caltech.edu/level5/ESSAYS/essays.html (une très bonne série d’articles)

        minute 18 : la conférencière évoque l’expérience « du soleil qui disparait ». Malheureusement, il ne s’agitpas une expérience de physique, Mais plutôt d’ une expérience de forum de geeks passionnés de physique….
        en plus inutile, il est facile de voir si la terre tourne autour du soleil où il est vraiment, ou bien autour du soleil où il était il y a huit minutes (dans le premier cas la terre suit une orbite képlérienne, dans le deuxième cas, un couple) se forme et l’orbite se modifie en permanence, à votre avis ?
        elle affirme que les masses déforment l’espace temps et que donc selon la distance à une masse, l’écoulement du temps va changer (d’accord) et les distances vont changer (premiere fois que j’entends dire ça et avec quoi pourrait-elle mesurer ces distances qui changent ?)
        avant la fin de la meme minute, elle saute de la gravité du soleil aux deux trous noirs et à leurs ondes gravitationnelles, quel gloubi-boulga ! c’est beaucoup trop rapide pour un coq à l’âne
        elle ferait mieux d’expliquer la distinction entre gravité et ondes gravitationnelles, au lieu de mettre une équation au tableau et de la décrire pendant trois minutes. Aucun bon vulgarisateur ne fait ça.
        Conclusion :  on a affaire à une mauvaise vulgarisatrice, sans aucun talent personnel. Comme est-elle arrivée là ?

        En effet je constate qu’une petite thésarde argentine se retrouve prof en Floride (si j’ai bien lu) puis propulsée dans les couloirs du LIGO ???
        La solution est peut-être donnée par le présentateur qui nous explique qu’elle a rencontré « quelqu’un » au cours de sa thèse …
        Qui est ce « quelqu’un » ? Il ne le dit pas mais ça devrait être facile à trouver, malgré une dissimulation évidente :
        Ce n’est pas sur la page Wikipedia de la dame, qui par ailleurs continue à se faire appeler Gonzalez…. alors qu’elle est probablement mariée ???
        Pour moi ça sent le gros piston

      • stefjourdan

        et vous devriez retravailler cette phrase « Vous avez toutes les réponses… » beaucoup moins littéraire que votre style habituel à la Stéphane Bern.

  8. akéla54

    Bonjour, je suis un néophyte qui s’interroge sur cette même question de l’instantaneité (ou non) de l’action de la gravitation et qui tombe sur votre long argumentaire qui rejoint mes intuitions sur la question.
    L’une de mes interrogations perplexes sur la théorie du « trampoline » concerne la direction uniforme des déformations représentées sur la trame « élastique » de l’Espace-Temps. Comment les tenants de cette théorie soutiennent-ils ce shéma d’une même direction de déformation du « trampoline » dans l’espace ? A quoi celà pourrait-il correspondre, un « grand Attracteur universel » ?
    Si ma remarque vous semble pertinente, merci de me le faire savoir.

    • stefjourdan

      Bonjour et merci de votre commentaire.Je ne comprends pas très bien votre question. Je ne sais pas si on peut parler de « théorie du trampoline ». C’est juste un modèle pédagogique, une analogie..Plusieurs auteurs font remarquer que l’expérience ne marche QUE parce que la gravité terrestre le permet (ce qui ne me semble pas réellement gênant, puisque c’est une analogie).. J’ajoute que par exemple EN ORBITE, le coup du trampoline ne marcherait pas. Pour revenir à votre question : que voulez-vous dire par « direction uniforme des déformations ». Sur le trampoline, l’enfoncement est toujours vers le bas (une direction qui « n’appartient pas » à l’espace en deux dimensions dans lequel les boules se meuvent : l’espace-plan. Par contre la pente de la toile, qui contraint le mouvement des boules 1 est bien un vecteur dans l’espace-plan 2 n’a pas toujours la même direction et a bien une liberté de 360°

      • akéla54

        Je reprécise mon interrogation sur le schéma du « trampoline ». Supposons l’observation de plusieurs objets célestes massifs, 1- pourquoi la direction de la déformation de la trame « espace-temps » serait la même pour chaqcun de ces objets ? 2- Quelle serait la cause commune cette direction unique de cette déformation, sachant que que chaque objet peut déformer son propre « espace-temps » de 360*360° directions possible ?
        L’image de la toile de trampoline déformée de manière uniforme me semble commode pour les schémas de la gravitation « généraliste » mais assez réductrice dans le concept.
        Bon, voilà vous ai-je paru plus claire ?
        Par avance, merci
        Akéla5

        « 1- pourquoi la direction de la déformation de la trame « espace-temps » serait la même pour chaqcun de ces objets ? »

        Je ne comprends toujours pas où vous voyez cette « même direction » pour différents objets, ce n’est pas clair du tout. Je vous ai déjà répondu que dans le modèle du trampoline, chaque point du trampoline a une pente avec intensité et direction différente, comme dans un champ de gravité, d’où l’intérêt du modèle.

        Donc il n’y a pas de « direction unique » et il n’y a pas ce que vous appelez « déformation uniforme » ????

        si vous voulez faites un schéma, (ou prenez une représentation existante et ajoutez-y vos annotations) et envoyez le moi à stephane.jourdan@gmail.com ?

  9. stefjourdan

    Q Concernant cet épineux problème de l’instantanéité de la gravitation, je suis tombé comme beaucoup sur des forums où les gens sont vraiment très sûr d’eux et ne cesse d’asséner que rien ne va plus vite que la lumière donc ceci cela. Bien entendu ce n’est pour moi pas suffisant mais le problème est que, hormis votre article, je ne suis pas arrivé à trouver d’information allant dans votre sens. Comme je l’ai dit, j’ai tendance à être d’accord avec vous mais je n’ai pas les compétences en physique et mathématiques pour pouvoir argumenter moi-même dans un sens ou l’autre alors je continue de m’interroger avec le peu que je sais (ou crois savoir).

    R Tout à fait : les gens « assènent » cette vérité sans réfléchir qu’elle n’est vraie que pour les masses et les ondes electro-magnétiques, jusqu’à preuve du contraire, et pas que les forums, aussi dans les « cours » en ligne, les sites officiels (j’en cite plusieurs).

    Q Voilà ce qui, de mon coté, me fait accepter l’idée d’une gravitation instantanée. D’après ce que j’ai saisi de la relativité générale, l’espace-temps est modifié par les masses. Il ne s’agit pas de quelque chose qui se déplace dans l’espace donc pourquoi ne pas envisager que l’espace-temps soit modifié en tout point de celui-ci par l’ensemble des masses qu’il contient et ce de manière instantanée même si cette notion fait appel au temps, lui-même modifié. Je me suis trouvé une sorte d’analogie: si on déplace un objet en translation (sans que l’objet ne soit en rotation) par l’action d’une force, on peut considérer que chaque point de cet objet a « subi » un déplacement qui s’est propagé de manière instantanée.

    R Dans le cas d’un objet, même très dur, il est en réalité élastique, et un mouvement imprimé à un bout ne peut se propager à vitesse infinie. Par exemple si on avait un rail très long, genre 100 km et qu’on le pousse brusquement à une extrémité (il faudrait être costaud) eh bien le mouvement ne pourrait pas se propager plus vite que le son dans ce métal, soit dans les 1000 km/h, donc l’autre extrémité du rail ne pourrait commencer à bouger qu’au bout d’un dixième d’heure, soit 6 mn. Dans la pratique dès que le bout poussé reçoit une accélération trop brusque le rail va se déformer sur le côté, ou si on l’en empêche, s’applatir etc. (sauf si c’est une poutre verticale en acier dont un avion a percuté le sommet avec beaucoup de kérosène, alors le kérosène en brûlant va ramollir l’acier sur toute la hauteur et le rail va s’effondrer instantanément car il perdra sa résistance au même moment sur toute la hauteur, bien entendu…non je blague)

    Peut-être que cet exemple n’est pas bon,

    C’est bien essayé, j’ai une idée similaire, mais sans faire appel à un objet. Il faudrait considérer le champ de gravitation, non pas comme un « effet » de la masse, mais comme la masse elle-même, car la gravitation est un mystère, si vous voulez, mais la masse elle même et la nature de l’espace vide aussi si on « va par là ». Si au lieu de voir trois mystères séparés vous vous mettez à considérer que c’est trois aspects d’une même réalité, alors il ne reste plus qu’un seul mystère : chaque atome se comporte, du point de vue « visible » (interaction avec les particules/ondes electromag) comme un objet local très petit (qu’il n’est pas bien entendu) dans l’espace mais au niveau de la gravité comme un objet très étendu. Un corps céleste, la lune par ex. se comporte comme donc une grosse masse locale que l’on « voit » mais aussi comme un champ de gravité qui ne l’oublions pas, n’est que la réunion des milliards de champs individuels de gravité de chacun de ses atomes.L’idée que les atomes, la masse, se déplace et qu’ENSUITE le champ de gravité a besoin de se propager vient de la séparation artificielle des deux/trois phénomènes. Si le champ de gravité EST la masse, le problème disparaît.

    Q peut-être que je n’ai pas les bonnes connaissances mais pour le moment, même si je suis prêt à accepter l’idée d’un effet non instantané, je n’ai pas trouvé d’explication suffisamment convaincante pour cela. De votre coté, avez-vous vu d’autres articles se rangeant à votre avis ?

    R je crois que j’ai cité certains articles qui posent la question. En dehors de ça, non, pas grand chose, mais je ne connais pas toutes les théories. Comme le montre mon article, il y a une grande confusion mentale, même à un haut niveau (apparent) de vulgarisation. Ces gens qui sont dans les labos de cosmologie etc ont fait des études longues et poussées certes, mais sur des sujets très précis, pas forcément de l’épistémologie. Ils ne savent pas tout et en plus ils ne sont pas forcément conscients des limites de leurs raisonnements. Dans la plupart des cas, au niveau cosmologique, ils travaillent sur des FORMULES mathématiques qui leur permettent de prévoir des choses. Donc ils passent surtout du temps à fignoler ces simulations pour qu’elles se rapprochent de la réalité. Par ailleurs ils évitent de parler de ce qui n’est pas compris : la rotation généralisée des corps céleste par exemple, juste pour bouter en touche….

  10. stefjourdan

    Bonjour Stéphane,

    merci de m’avoir répondu. Ci-dessous 2 liens sur lesquelles je suis tombé et auxquelles je faisais référence (il y en a sûrement d’autres):
    http://blog.slate.fr/globule-et-telescope/2012/12/27/la-gravite-se-deplacerait-bien-a-la-vitesse-de-la-lumiere
    http://www.rtflash.fr/vitesse-gravite-einstein-avait-raison/article

    Réponse : Quant au deusième article, j’avais déjà fait des commentaires en bas…(il s’agit du cas classique de confusion)

    J’avais visité également le premier, très journalistique. La phrase qui passe des OG à la gravité (« d’autant que …. »)ne semble pas montrer une confusion totale mais ce n’est pas suffisemment encourageant par rapport aux connaissances de l’auteur.

    1 je ne crois pas que des mesures sur « les marées » ou « les éclipses » puissent nous renseigner sur la vitesse de propagation de la gravitation, je demande à voir plus d’explications.

    2 dans le film Gravity, les protagonistes ne dérivent pas dans « l’immensité de l’espace », ils sont en orbite. Il est vrai que dans ces conditions (chute libre) les expérimentateurs se retrouvent apparemement en micro-gravité. En réalité ils sont bel et bien dans le champs gravitaire de la terre et sont en accélération constante sur une trajectoire Keplerienne…

  11. Vincent

    Bonjour,
    merci pour cette clarification. Je suis plutôt de votre avis mais que penser de certains articles indiquant que des équipes de recherche avaient prouvé expérimentalement que le gravité se propage bien à la vitesse de la lumière ?

    • stefjourdan

      Bonjour Vincent

      merci à vous pour cette appréciation

      il y a deux sortes d’articles qui disent ça

      1 tout ceux qui confondent (comme je le montre) ondes gravitationnelles et gravité, souvent de façon très caricaturale; ils parlent donc des Ondes G, en réalité des
      vibrations de l’espace temps qui sont d’ORIGINE gravitationelle, mais ne sont pas de la gravité

      2 des articles qui étudient réellement la vitesse de propagation de la gravité, par ex ceux qui traitaient de la déviation de la lumière d’étoiles lointaines (je crois) au moment de leur passage près de (? une planète, le soleli, je ne sais plus)

      si la gravité de la planète ‘déviante » se propage instantanément ou à C on devrait observer des effets différents (?),à voir au coup par coup dans ce deuxième cas

      je rappelle qu’il y a des expériences beaucoup plus simples qui pourraient montrer un « délai de la gravité » : les planètes, au fur et à mesure qu’on s’écarte du soleil, devrait subir sa gravité avec un délai, très court pour les planètes proches, beaucoup plus long pour les planètes lontaines, pluton est combien de minutes (heures) lumière du S ?

      en écrivant cela, je me rends compte que si le délai est proportionnel à la distance, alors l’angle d’abberation de la gravité devrait être le même (triangles proportionnels) pour toutes les planètes quel que soit leur distance ???

      Bien à vous

Laissez une réponse à stefjourdan Annuler la réponse.